QUESTIONS Flashcards

1
Q

A 42-year-old woman is evaluated in the emergency department. She has a 2-day history of nonexertional chest pain. The pain is sharp, substernal, and worse when lying down or with deep breaths. She denies shortness of breath. Her symptoms were preceded by a recent upper respiratory tract infection. On physical examination, temperature is 37.9°C (100.3°F), blood pressure is 165/90 mm Hg, pulse rate is 102/min, respiration rate is 18/min, and oxygen saturation is 96% on ambient air. The cardiopulmonary examination is normal as is the remainder of the physical examination. An electrocardiogram is shown :. Echocardiogram shows a small pericardial effusion. Which of the following is the most appropriate initial therapy?

A

The most appropriate initial therapy is ibuprofen. Acute pericarditis is the most likely diagnosis. Viral infection is the most common known cause of acute pericarditis. Diagnosis is most often made by confirming two of three classic findings: chest pain, often with a pleuritic component; friction rub; and diffuse ST-segment elevation on electrocardiography (ECG). This patient has chest pain and diffuse ST-segment elevation on ECG. A pericardial friction rub is virtually pathognomonic of acute pericarditis. It is best auscultated at the left lower sternal border during suspended respiration while the patient is leaning forward. The classic rub has three components and can be squeaky, scratchy, or swooshing. It is often transient. Although a pericardial friction rub is a highly specific sign of acute pericarditis, it is not very sensitive and its absence does not rule out the diagnosis. In acute pericarditis, epicardial inflammation causes upwardly concave ST-segment elevation in all leads except aVR. PR-segment depression in the limb leads (particularly lead II) or precordial leads and elevation in lead aVR may accompany ST-segment elevation. PR-segment shifts are highly specific but not sensitive findings for acute pericarditis. Echocardiography can detect very small pericardial effusions that may help with the diagnosis of pericarditis. More importantly, echocardiography can detect cardiac tamponade and heart failure that may complicate acute pericarditis. Appropriate initial therapy for acute pericarditis is either aspirin or an a nonsteroidal anti-inflammatory drug (NSAID), such as ibuprofen. Chest pain resulting from acute pericarditis usually resolves within 24 hours of treatment with anti-inflammatory medications. Nonrandomized and nonblinded studies support the use of colchicine as an alternative first-line agent or an adjunctive treatment for acute pericarditis. In the absence of a specific indication for their use, a glucocorticoid, such as prednisone, should only be used in refractory cases of acute pericarditis or in patients with contraindications to aspirin, NSAIDs, and colchicine. Glucocorticoids increase the risk of recurrent pericarditis. The nature of the chest pain and the findings of PR-segment depression on ECG and a small effusion on echocardiography are most consistent with acute pericarditis and make acute coronary syndrome or pulmonary embolism unlikely. Therefore, heparin and clopidogrel are not indicated.

How well did you know this?
1
Not at all
2
3
4
5
Perfectly
2
Q

A 60-year-old man is evaluated because of a 3-month history of intermittent chest pain. He has occasional substernal chest pressure when he exercises at the gym and occasionally after he eats a spicy meal. The pressure is not consistently relieved with rest and is occasionally relieved with antacid. He has no associated symptoms of shortness of breath, dizziness, or diaphoresis. His medical history includes hypertension and hyperlipidemia. Medications are lisinopril and pravastatin. On physical examination, blood pressure is 128/80 mm hg, pulse rate is 84/min, and respiration rate is 16/min. Findings on cardiovascular examination are normal. The electrocardiogram is shown Which of the following is the most appropriate diagnostic test to evaluate the patient’s chest pain?n

A

The most appropriate test to evaluate the patient’s chest pain is exercise electrocardiography. A variety of noninvasive stress tests are available to determine whether a patient with cardiovascular symptoms has coronary artery disease (CAD). The decision to perform a specific test is based on the pretest probability of CAD, the patient’s ability to exercise, findings on resting electrocardiography (ECG), and comorbid conditions, such as reactive airways disease, that may influence the choice of a pharmacologic stress agent. Stress testing is most useful in patients with an intermediate pretest probability of CAD. For patients with a low pretest probability of CAD, stress testing is not useful because an abnormal test result is likely a false-positive finding and a normal test result only confirms the low pretest probability of CAD. For patients with a high pretest probability of CAD, stress testing is not useful to diagnose CAD and empiric medical therapy should be initiated. In this setting, a normal stress test result would likely be a false-negative finding and an abnormal stress test result would only confirm a high pretest probability of CAD. Exercise ECG testing is the standard stress test for the diagnosis of CAD in patients with normal baseline ECG findings. If abnormalities that limit ST-segment analysis are present (left bundle branch block, left ventricular hypertrophy, paced rhythm, Wolff-Parkinson-White pattern), the results may be difficult to interpret. In patients with abnormalities on resting ECG that impair the ability to interpret ST-segment changes, imaging increases diagnostic accuracy and ability to determine the site and extent of ischemia. Exercise is preferred to pharmacologic stressors because it provides a gauge of functional capacity and a contextual understanding of symptoms as well as a record of the hemodynamic response to exercise. For patients who cannot exercise because of physical limitations or physical deconditioning, pharmacologic stressors can be used. These agents, recommended if the patient cannot achieve at least 5 metabolic equivalents, increase myocardial contractility and oxygen demand (dobutamine) or induce regional hypoperfusion through coronary vasodilation (adenosine, dipyridamole, and regadenoson). This patient has atypical chest pain (substernal pressure caused by exercise but not relieved with rest) and normal findings on ECG. The most appropriate diagnostic test to evaluate the patient’s chest pain is exercise electrocardiography. Because he is a candidate for the preferred type of stress test, alternative methods, including imaging with echocardiography or pharmacologic stress testing with dobutamine, are not indicated. Cardiac catheterization would not be an appropriate intervention given the patient’s intermediate risk for cardiovascular disease. It might be appropriate if he had a high pretest probability of coronary artery disease or he had specific findings of coronary occlusion on stress testing.

How well did you know this?
1
Not at all
2
3
4
5
Perfectly
3
Q

A 24-year-old man is evaluated for a 6-month history of episodic substernal chest pain. Episodes occur four to five times per week and are accompanied by palpitations and sweating. They resolve spontaneously after approximately 30 minutes. His symptoms are unrelieved with antacids, can occur at rest or with exertion, and are nonpositional. There are no specific precipitating factors. Lipid levels were obtained last year and were normal. The patient is a nonsmoker. He has no personal or family history of coronary artery disease, diabetes mellitus, hyperlipidemia, or hypertension. He is not taking any medications.

On physical examination, vital signs are normal. He has no cardiac murmurs and no abdominal pain. Complete blood count, serum thyroid-stimulating hormone level, and electrocardiogram are all normal.

Which of the following is the most appropriate management of this patient?

A

The most appropriate management of this patient is to prescribe a selective serotonin reuptake inhibitor. Panic disorder is a syndrome characterized by sudden panic attacks with acute onset of somatic symptoms that may include chest pain, palpitations, sweating, nausea, dizziness, dyspnea, and numbness. These symptoms usually last from 5 to 60 minutes. Approximately 50% of patients with panic disorder also have associated agoraphobia, with fears of being in crowds or in places from which escape would be difficult. Diagnosis is based on clinical descriptors and setting, but care should be taken to consider underlying medical disorders, such as cardiac disease, thyroid disease, or pheochromocytoma, particularly in patients who are at increased risk for one of these disorders. However, extensive testing is not necessary in most patients with a characteristic presentation and normal findings on physical examination and basic laboratory studies. Treatment options for panic disorder include medication and psychotherapy. Cognitive behavioral therapy (CBT) has been shown to be the most effective psychotherapeutic intervention in controlled trials. Selective serotonin reuptake inhibitors and serotonin-norepinephrine reuptake inhibitors have been shown to be effective. Panic disorder that is severe or refractory appears to be most amenable to a combination of CBT and pharmacotherapy compared with either treatment alone.

This patient has classic symptoms of panic disorder and no cardiac risk factors. It would be inappropriate to order further cardiac testing in the setting of a normal electrocardiogram and classic symptoms. This patient’s symptoms are also atypical for gastroesophageal reflux disease, and so empiric proton pump inhibitor therapy would be inappropriate.

How well did you know this?
1
Not at all
2
3
4
5
Perfectly
4
Q

A 70-year-old man is evaluated for sharp left-sided pleuritic chest pain and shortness of breath that began suddenly 24 hours ago. The pain has been persistent over the last 24 hours and does not worsen or improve with exertion or position. The patient’s history is significant for a 50-pack-year smoking history and severe chronic obstructive pulmonary disease, although he is currently a nonsmoker. Medications are ipratropium and albuterol.

On physical examination, temperature is normal, blood pressure is 128/80 mm Hg, pulse rate is 88/min, and respiration rate is 18/min. Oxygen saturation on ambient air is 89%. The trachea is midline. Lung examination shows hyperresonance, decreased chest wall expansion, and decreased breath sounds on the left. Cardiac examination shows distant heart sounds but no extra heart sounds.

Which of the following is the most appropriate diagnostic test to perform next?

A

This patient should undergo chest radiography. He has severe chronic obstructive pulmonary disease (COPD) and findings that are consistent with spontaneous secondary pneumothorax. These findings include sudden, sharp, nonradiating pleuritic chest pain and shortness of breath with hyperresonance, decreased breath sounds, and decreased chest wall expansion on the side of the pneumothorax in a patient with underlying lung disease. Pneumothorax should be considered in any patient who has sudden onset of pleuritic chest pain and dyspnea. The diagnostic test of choice if pneumothorax is suspected is an upright chest radiograph. Findings on chest radiograph include separation of the parietal and visceral pleura by a collection of gas and the absence of vessels in this space. This patient is at increased risk for pneumothorax because of his COPD. Pneumothorax occurring in patients without known lung disease or a clear precipitating cause is termed primary spontaneous pneumothorax (PSP). PSP tends to occur more often in men, smokers, and those with a family history of PSP. The clinical presentation is similar in both primary or secondary pneumothorax.

Chest computed tomography (CT) also can be used to diagnose pneumothorax. Chest CT may be more sensitive than a chest radiograph in delineating smaller collections of gas in the pleural space and providing more information about the pulmonary parenchyma and pleura. However, plain film radiography remains the initial test of choice for most patients, and chest CT should be reserved for cases when the chest radiograph does not provide information to guide further treatment or evaluation.

The patient’s history and physical examination are classic for pneumothorax, and his pain descriptors do not strongly suggest ischemia or other primary cardiovascular disease. Electrocardiogram and echocardiogram, which are the tests of choice to evaluate ischemic heart disease, valvular heart disease, or cardiomyopathy, would not be the first diagnostic tests of choice for suspected pneumothorax.

How well did you know this?
1
Not at all
2
3
4
5
Perfectly
5
Q

A 50-year-old woman is evaluated for a 1-year history of recurrent left-sided chest pain. The pain is poorly localized and nonexertional and occurs in 1-minute episodes. There is no dyspnea, nausea, or diaphoresis associated with these episodes. The patient has not had dysphagia, heartburn, weight change, or other gastrointestinal symptoms. She has no other medical problems and does not smoke cigarettes.

On physical examination, vital signs are normal. The patient’s chest pain is not reproducible by palpation. The cardiac examination is unremarkable, as is the remainder of the physical examination.

Results of a lipid panel, a fasting plasma glucose test, and a chest radiograph are normal. An echocardiogram shows a normal ejection fraction, with no wall motion abnormalities. Results of an exercise stress test are normal.

Which of the following is the most appropriate next step in management?

A

This patient should receive twice-daily proton pump inhibitor (PPI) therapy for 8 to 10 weeks. Pain associated with gastroesophageal reflux can mimic ischemic chest pain. A cardiac cause should be carefully assessed and excluded in all patients with chest pain. This patient has nonanginal chest pain, no additional risk factors, and normal findings on exercise stress testing. Therefore, the likelihood of a cardiac cause of chest pain is low. Randomized controlled trials have shown that a therapeutic trial of twice-daily PPI treatment is effective in 50% to 60% of patients with noncardiac chest pain, indicating that gastroesophageal reflux disease is the underlying cause.

If the PPI trial is unsuccessful, further evaluation with endoscopy (to detect erosive esophagitis or achalasia), manometry (to detect esophageal motility disorders such as diffuse esophageal spasm), and ambulatory pH recording (to detect refractory reflux) would be reasonable.

Musculoskeletal chest pain has an insidious onset and may last for hours to weeks. It is most recognizable when it is sharp and localized to a specific area of the chest; however, it can also be poorly localized. The pain may be worsened by turning, deep breathing, or arm movement. Chest pain may or may not be reproducible by chest palpation (pain reproduced by palpation does not exclude ischemic heart disease), and findings on cardiovascular examination are often normal. This patient does not have the typical features of musculoskeletal chest pain. Therefore, treatment with an nonsteroidal anti-inflammatory drug is not the most appropriate first step in management.

How well did you know this?
1
Not at all
2
3
4
5
Perfectly
6
Q

A 65-year-old man is evaluated because of chronic angina. He has a 10-year history of symptomatic coronary artery disease. The diagnosis was confirmed with an exercise stress test. Results of the test showed no high-risk features. His estimated left ventricular ejection fraction by echocardiography at that time was 56%. He occasionally has chest pain after walking four blocks. The pain is relieved by taking one sublingual nitroglycerin or by resting. His exercise capacity has not diminished, and the frequency, character, and duration of the pain have not changed. He denies shortness of breath, orthopnea, or paroxysmal nocturnal dyspnea. Current medications include simvastatin, aspirin, metoprolol, and sublingual nitroglycerin.

On examination, blood pressure is 122/82 mm Hg, pulse rate is 68/min, respiratory rate is 16/min, and body mass index is 27. There is no jugular venous distention, and there are no murmurs, gallops, rubs, or pulmonary crackles or peripheral edema.

Which of the following is the most appropriate management?

A

Continuing the current management is the most appropriate action. Several large trials have compared revascularization with optimal medical therapy. These studies found that revascularization combined with aggressive medical therapy was not superior to aggressive medical therapy alone in reducing death or myocardial infarction. Based on these studies, current guidelines recommend reserving coronary angiography and revascularization for patients who continue to have symptoms despite optimal medical therapy, patients who are unable to tolerate the side effects of medications, and those who have high-risk findings on noninvasive testing. This patient’s pattern of angina is stable, he is tolerating his medications, and his original stress test showed no high-risk features. Therefore, a more aggressive intervention such as coronary angiography is not indicated as there would not be an expected improved outcome.

Routine follow-up electrocardiography, exercise stress testing (or other noninvasive imaging studies), and echocardiography are not indicated in patients with chronic stable angina. Although careful and frequent follow-up is indicated, disease assessment is conducted with a detailed history and physical examination. The history should focus on changes in physical activity and the frequency, severity, or pattern of chest pain. Reasonable laboratory monitoring includes periodic measurement of lipids and blood glucose levels. Follow-up electrocardiography should be considered when there are medication changes that could affect cardiac conduction; there is a change in the severity, frequency, or pattern of angina; symptoms that suggest dysthymia occur; or syncope develops. There are no indications for an electrocardiogram, an exercise stress test, or echocardiography.

How well did you know this?
1
Not at all
2
3
4
5
Perfectly
7
Q

A 60-year-old man is evaluated because of a 3-month history of intermittent chest pain. He has occasional substernal chest pressure when he exercises at the gym and occasionally after he eats a spicy meal. The pressure is not consistently relieved with rest and is occasionally relieved with antacid. He has no associated symptoms of shortness of breath, dizziness, or diaphoresis. His medical history includes hypertension and hyperlipidemia. Medications are lisinopril and pravastatin.

On physical examination, blood pressure is 128/80 mm hg, pulse rate is 84/min, and respiration rate is 16/min. Findings on cardiovascular examination are normal.

The electrocardiogram is shown :.

Which of the following is the most appropriate diagnostic test to evaluate the patient’s chest pain?

A

The most appropriate test to evaluate the patient’s chest pain is exercise electrocardiography.

A variety of noninvasive stress tests are available to determine whether a patient with cardiovascular symptoms has coronary artery disease (CAD). The decision to perform a specific test is based on the pretest probability of CAD, the patient’s ability to exercise, findings on resting electrocardiography (ECG), and comorbid conditions, such as reactive airways disease, that may influence the choice of a pharmacologic stress agent. Stress testing is most useful in patients with an intermediate pretest probability of CAD. For patients with a low pretest probability of CAD, stress testing is not useful because an abnormal test result is likely a false-positive finding and a normal test result only confirms the low pretest probability of CAD. For patients with a high pretest probability of CAD, stress testing is not useful to diagnose CAD and empiric medical therapy should be initiated. In this setting, a normal stress test result would likely be a false-negative finding and an abnormal stress test result would only confirm a high pretest probability of CAD.

Exercise ECG testing is the standard stress test for the diagnosis of CAD in patients with normal baseline ECG findings. If abnormalities that limit ST-segment analysis are present (left bundle branch block, left ventricular hypertrophy, paced rhythm, Wolff-Parkinson-White pattern), the results may be difficult to interpret. In patients with abnormalities on resting ECG that impair the ability to interpret ST-segment changes, imaging increases diagnostic accuracy and ability to determine the site and extent of ischemia. Exercise is preferred to pharmacologic stressors because it provides a gauge of functional capacity and a contextual understanding of symptoms as well as a record of the hemodynamic response to exercise. For patients who cannot exercise because of physical limitations or physical deconditioning, pharmacologic stressors can be used. These agents, recommended if the patient cannot achieve at least 5 metabolic equivalents, increase myocardial contractility and oxygen demand (dobutamine) or induce regional hypoperfusion through coronary vasodilation (adenosine, dipyridamole, and regadenoson).

This patient has atypical chest pain (substernal pressure caused by exercise but not relieved with rest) and normal findings on ECG. The most appropriate diagnostic test to evaluate the patient’s chest pain is exercise electrocardiography. Because he is a candidate for the preferred type of stress test, alternative methods, including imaging with echocardiography or pharmacologic stress testing with dobutamine, are not indicated.

Cardiac catheterization would not be an appropriate intervention given the patient’s intermediate risk for cardiovascular disease. It might be appropriate if he had a high pretest probability of coronary artery disease or he had specific findings of coronary occlusion on stress testing.

How well did you know this?
1
Not at all
2
3
4
5
Perfectly
8
Q

A 55-year-old woman is evaluated for symptoms of sharp, localized, left-sided chest pain for the last 3 weeks. The pain is unrelated to exertion and is associated with mild dyspnea and fatigue. Typically it lasts for 5 to 10 minutes and abates spontaneously. The pain is not pleuritic, positional, or related to eating. She has hypertension and hypercholesterolemia. Her father had a myocardial infarction at 54 years of age. Daily medications are hydrochlorothiazide, simvastatin, and aspirin.

On physical examination, blood pressure is 135/78 mm Hg, pulse rate is 78/min, and respiration rate is 14/min. Cardiac auscultation shows S4 but is otherwise normal, as is the remainder of her physical examination.

Electrocardiogram shows sinus rhythm, with a heart rate of 75/min and no ST-segment or T-wave abnormalities.

Which of the following is the most appropriate diagnostic study?

A

This patient should undergo exercise electrocardiography (ECG). She has several risk factors for coronary artery disease (CAD), including hypertension, hypercholesterolemia, and a family history of premature CAD. Her symptoms are not typical of angina (her chest discomfort is localized, sharp, and not reproducible with exertion), and the resting ECG is normal. Because this patient has multiple risk factors and atypical symptoms, the pretest probability that CAD is the cause of her symptoms is intermediate. The results of exercise ECG testing, whether normal or abnormal, will significantly affect the posttest probability of CAD. Exercise testing is recommended as the initial test in patients with an intermediate pretest probability of CAD based on age, sex, and symptoms, including patients with right bundle branch block or ST-segment depression of less than 1 mm at baseline. In addition, the results of the exercise ECG test will provide prognostic information about the risk of death and myocardial infarction on the basis of exercise duration, angina, and the magnitude of ST-segment depression.

Coronary angiography is not appropriate for this patient because the pretest probability of CAD is intermediate, which is too low to warrant immediate coronary angiography as the initial diagnostic test.

Although exercise ECG testing has been found to have lower specificity and a higher false-positive rate in women than in men, the routine use of exercise testing with echocardiography to assess left ventricular regional wall motion or perfusion imaging is not recommended for either women or men in the absence of baseline ECG abnormalities. Although echocardiography increases the sensitivity of the ECG results, the use of stress echocardiography as the initial test has not been found to reduce cardiovascular events compared with exercise ECG testing alone.

Pharmacologic stress testing is not indicated because this patient is physically able to exercise. Pharmacologic agents include dobutamine (which increases heart rate and myocardial contractility) and vasodilators (which cause relative increases in coronary blood flow in myocardial regions that are not supplied by stenotic vessels). Exercise is preferred over pharmacologic treatment because of the additional diagnostic and prognostic information provided by exercise testing.

How well did you know this?
1
Not at all
2
3
4
5
Perfectly
9
Q

A 68-year-old man is evaluated for exertional chest pain of 3 months’ duration. He describes the chest pain as midsternal pressure without radiation that occurs with walking one to two blocks and resolves with rest or sublingual nitroglycerin. No symptoms have occurred at rest. His medical history is significant for myocardial infarction 3 years ago, hypertension, and hyperlipidemia. Medications are aspirin, metoprolol, simvastatin, isosorbide dinitrate, and sublingual nitroglycerin as needed for chest pain.

On physical examination, temperature is normal, blood pressure is 150/85 mm Hg, pulse rate is 80/min, and respiration rate is 12/min. The lungs are clear. Cardiac examination shows normal S1 and S2 with no extra heart sounds or murmurs. The remainder of the examination is unremarkable.

Electrocardiogram shows normal sinus rhythm, no left ventricular hypertrophy, no ST- or T-wave changes, and no Q waves.

Which of the following is the most appropriate management?

A

In this patient with coronary artery disease and continuing angina, the medical therapy should be optimized by increasing the dosage of β-blocker. Physical examination is notable for blood pressure and heart rate that would allow further up-titration of the β-blocker dosage. Complete β-blockade typically results in a resting pulse rate of approximately 55 to 60/min. Therefore, the pulse rate of 80/min suggests that the dosage of metoprolol should be increased. β-blockers are particularly effective antianginal medications because they decrease heart rate, myocardial contractility, and systemic blood pressure, thereby lowering myocardial oxygen demand.

Calcium channel blockers are first-line antianginal therapy in patients with contraindications to β-blockers. In patients with continuing angina despite optimal dosages of β-blocker and nitrates, a calcium channel blocker may be added. A calcium channel blocker such as diltiazem is not indicated in this patient because his dosage of metoprolol is not yet optimal.

Ranolazine should be considered only in patients who remain symptomatic despite optimal dosages of β-blockers, calcium channel blockers, and nitrates. Ranolazine decreases angina symptoms but is significantly more expensive and less effective than the usual antianginal medications.

Coronary angiography may be indicated in a patient who is receiving maximal medical therapy with continued symptoms of angina that affect quality of life. Referral for coronary angiography is not indicated for this patient because he is not currently receiving optimal medical therapy.

How well did you know this?
1
Not at all
2
3
4
5
Perfectly
10
Q

A 62-year-old man with coronary artery disease is evaluated for angina. He was diagnosed 4 years ago, and since that time, his symptoms have been well controlled with metoprolol and isosorbide mononitrate. He had exertional angina 8 months ago. His dosages of metoprolol and isosorbide were increased and long-acting diltiazem was added, with improved control of his symptoms. He has had increasing symptoms over the last 2 months and now requires daily sublingual nitroglycerin for relief of angina during exercise. He has not had any episodes of angina at rest. His medical history is significant for hyperlipidemia treated with atorvastatin.

On physical examination, the patient is afebrile, blood pressure is 110/60 mm Hg, pulse rate is 55/min, and respiration rate is 12/min. Results of cardiopulmonary examination are unremarkable, as are the remainder of the findings of the physical examination.

Electrocardiogram shows no acute ischemic changes.

Which of the following should be the next step in this patient’s management?

A

Coronary angiography is the most appropriate option in this patient who has had continued symptoms of angina despite optimal medical therapy. Although it has been shown that a routine strategy of coronary angiography and revascularization provides no benefit compared with optimal medical therapy in patients with chronic stable angina, coronary angiography may be of benefit in patients who are highly symptomatic despite optimal medical therapy, such as this patient. Coronary angiography allows direct evaluation of the coronary anatomy, with possible percutaneous coronary intervention or surgical revascularization if indicated. Coronary revascularization has been shown to be beneficial in patients with chronic stable angina and the following conditions: angina pectoris that is refractory to medical therapy; a large area of ischemic myocardium and high-risk criteria on stress testing; high-risk coronary anatomy, including left main coronary artery stenosis or three-vessel disease; and significant coronary artery disease with reduced left ventricular systolic function. In appropriately selected patients, revascularization, with either percutaneous coronary intervention or coronary artery bypass grafting surgery, has been shown to reduce angina, increase longevity, and improve left ventricular performance.

Exercise treadmill stress testing would not be useful in the management of this patient because it would only confirm the known diagnosis of coronary artery disease. Results of an exercise stress test would not influence therapeutic decisions.

Although β-blockers and nitrates are effective antianginal medications, the patient is receiving near-maximal doses of both drugs, as indicated by his pulse rate, which shows effective β-blockade, and his blood pressure, which likely would not tolerate an increase in the dosage of either medication.

How well did you know this?
1
Not at all
2
3
4
5
Perfectly
11
Q

A 68-year-old man is evaluated in the emergency department because of a 2-day history of intermittent chest pain. The pain is substernal, is not related to activity, and lasts less than 15 minutes. His medical history includes hypertension, hyperlipidemia, and type 2 diabetes. His medications include aspirin, metoprolol, lisinopril, simvastatin, and metformin. In the emergency department he received a dose of liquid antacid, and his chest pain partially resolved.

On physical examination, he is afebrile, blood pressure is 130/80 mm Hg, pulse rate is 70/min, respiration rate is 18/min, and oxygen saturation is 98% on ambient air. Results of cardiopulmonary examination are normal.

The electrocardiogram shows minor T-wave abnormalities. Troponin T measurement is less than 0.01 ng/mL (0.01 µg/L).

Which of the following is the most appropriate management of this patient’s chest pain?

A

The most appropriate management of this patient’s chest pain is admission to the telemetry unit and ongoing assessment with serial electrocardiograms and troponin measurements. Based on this patient’s age and the substernal nature of his chest pain, there is at least an intermediate likelihood of an acute coronary syndrome. The decision to hospitalize a patient with chest pain is challenging. The goal is to identify patients with life-threatening disease who require immediate attention while minimizing unnecessary evaluation and treatment in others. A rapid clinical determination of the likelihood of an acute coronary syndrome is the essential first task and should guide the admission decision. In addition, the physician should consider the likelihood of short-term adverse outcomes, including death and nonfatal myocardial infarction, in patients with acute coronary syndrome. When acute coronary syndrome is suspected, the patient should be admitted for evaluation and management. Low-risk patients can be further stratified with stress testing.

At this point, the patient does not have an indication for coronary angiography and acute intervention. If further chest pain develops in association with ST-segment or T-wave changes on electrocardiography or elevated cardiac enzyme levels and the patient is considered high risk according to his TIMI (Thrombolysis in Myocardial Infarction) score, cardiac angiography would be reasonable.

The most common gastrointestinal cause of chest pain is gastroesophageal reflux disease (GERD). Although the pain associated with GERD is often described as burning, it can mimic angina and may be relieved by nitroglycerin. It generally is worsened with bending over or recumbency and is relieved with antacids, histamine-2 blockers, or proton pump inhibitors. Because acute coronary syndrome is a life-threatening condition, this diagnosis must be addressed first. In addition, the preferred initial diagnostic test for GERD is a therapeutic trial of a proton pump inhibitor, not esophageal pH monitoring.

Cardiac stress testing can be highly valuable in identifying significant coronary insufficiency, and it would be the test of choice to diagnose stable angina. However, in the acute setting, stress testing is contraindicated in a patient with possible acute coronary syndrome. Once the patient is stabilized and acute coronary syndrome is excluded, stress testing can be used to further stratify risk in a low- or intermediate-risk patient. In this patient, the preferred cardiac stress test is an exercise stress test, not a pharmacologic stress test with nuclear imaging.

How well did you know this?
1
Not at all
2
3
4
5
Perfectly
12
Q

A 58-year-old woman is evaluated in the emergency department for chest pain, diaphoresis, and shortness of breath of 4 hours’ duration. Three years ago she was diagnosed with a non–ST-elevation myocardial infarction and was treated medically. Additional medical history includes type 2 diabetes and hypertension. Her current medications are aspirin, lisinopril, atorvastatin, and glargine insulin.

On physical examination, she is afebrile, blood pressure is 125/60 mm Hg, pulse rate is 48/min, respiratory rate is 18/min, and oxygen saturation is 98% on ambient air. Cardiac examination shows no jugular venous distention, and the lungs are clear. An S4 is present.

Electrocardiogram shows ST-segment elevation and T-wave inversions in leads II, III, and aVF. The initial troponin T measurement is elevated.

The nearest hospital capable of percutaneous coronary intervention is more than 2 hours away. The patient has no contraindication to thrombolytic therapy.

Which of the following is the most appropriate initial management for this patient?

A

The most appropriate initial management of this patient consists of aspirin, heparin, intravenous nitroglycerin, and thrombolytic therapy. This patient has evidence of an ST-elevation myocardial infarction (STEMI) with acute ST-segment changes present in the inferior leads and an elevated troponin T level. The treatment of choice for STEMI is reperfusion therapy. Reperfusion for patients with STEMI can be achieved with thrombolytic therapy or primary percutaneous coronary intervention (PCI). Many patients with STEMI in the United States present to non–PCI-capable hospitals; as a result, thrombolytic therapy and transfer for primary PCI are the available treatment options. The time to achieve balloon inflation is a major determinant of the benefits of PCI versus thrombolytic therapy. If PCI must be delayed, then thrombolytic therapy should be considered. Observational data from community hospitals within the United States have found that fewer than 5% of patients achieve the guideline-suggested door-to-balloon time of less than 90 minutes. A projected 2-hour delay to a PCI facility makes thrombolytic therapy the best option for this patient.

β-Blockers reduce mortality and should be given to all patients with acute coronary syndrome except those with heart failure, systolic blood pressure of less than 90 mm Hg, bradycardia (<50/min), or second-degree atrioventricular block. Because this patient’s heart rate is 48/min, metoprolol should not be administered. Treatment with heparin, clopidogrel, pravastatin, and intravenous nitroglycerin without thrombolytic therapy does not address the need for immediate myocardial reperfusion and is not adequate treatment for this patient.

How well did you know this?
1
Not at all
2
3
4
5
Perfectly
13
Q

A 52-year-old woman was evaluated in the emergency department because of acute onset of dyspnea while shoveling snow this morning. The dyspnea resolved within 2 minutes of rest but recurred an hour later while she was watching television. Over the previous 10 days she has had several similar episodes of dyspnea with mild exertion, such as walking upstairs, and also at rest. She has no chest pain, palpitations, or orthopnea. She has a 15-year history of type 2 diabetes, hyperlipidemia, and hypertension treated with aspirin, metformin, chlorthalidone, ramipril, and rosuvastatin.

On physical examination, temperature was 37°C (98.6°F), blood pressure was 110/70 mm Hg, pulse rate was 80/min, respiratory rate was 18/min, and oxygen saturation was 96% on ambient air. There was no jugular distention, normal cardiac sounds were present without extra sounds or murmurs, and the lungs were clear to auscultation.

The initial electrocardiogram showed ST-segment changes. The first troponin I level was 0 ng/mL (0 µg/L).

An hour after admission to the emergency department, she had an episode of acute dyspnea. A repeat electrocardiogram at this time is shown :. Repeat troponin level is 0.8 ng/mL (0.8 µg/L).

A

The most appropriate next test is cardiac catheterization. Although chest pain is the most common presenting symptom for an acute coronary syndrome, women and patients with diabetes are more likely to have atypical angina symptoms, such as fatigue, dyspnea, and nausea. Acute coronary syndromes include unstable angina, non–ST-elevation myocardial infarction (NSTEMI), and ST-elevation myocardial infarction (STEMI). Patients with ischemic chest pain and STEMI benefit from reperfusion therapy, either thrombolytic therapy or primary angioplasty. Patients with NSTEMI or unstable angina are a heterogeneous group and require risk stratification (determination of their risk of death or nonfatal myocardial infection) to direct therapy. Although patients with unstable angina may have similar electrocardiographic findings to those with NSTEMI, they can be differentiated by the lack of elevation in serum cardiac biomarkers. This patient with recurrent unprovoked dyspnea, an evolving electrocardiogram showing ischemic changes in the anterolateral leads (ST-segment depression and T-wave inversion), and rising troponin level is at high risk for death or nonfatal myocardial infarction (5 points on the TIMI (Thrombolysis in Myocardial Infarction) risk stratification score). This patient will benefit from an early invasive approach that includes coronary angiography and subsequent revascularization (percutaneous coronary intervention or surgical revascularization). Most contemporary trials evaluating these two alternatives show a benefit for early angiography and revascularization (invasive approach).

An echocardiogram may show localized wall motion abnormalities associated with acute ischemia, but this finding does not alter the need for early intervention with cardiac catheterization and revascularization. An exercise stress test is contraindicated in patients with unstable ischemic heart disease. Repeating the troponin level will add no useful additional information to guide the management of this patient.

How well did you know this?
1
Not at all
2
3
4
5
Perfectly
14
Q

A 78-year-old man is evaluated in the emergency department because of chest pain. He describes left substernal discomfort that began approximately 8 hours ago. He reports no similar episodes of chest pain. Medical history is significant for hypertension and a 30-pack-year history of ongoing tobacco use. His only medication is amlodipine.

On physical examination, the patient is afebrile, blood pressure is 130/80 mm Hg, pulse rate is 72/min, and respiration rate is 12/min. There is no jugular venous distention, the lung fields are clear, and cardiac examination shows a normal S1 and S2 without murmurs. No peripheral edema is present.

The initial troponin I level is 26 ng/mL (26 µg/L). Laboratory findings are otherwise normal.

Electrocardiogram shows sinus rhythm of 70/min and 2-mm ST-segment elevation in leads II, III, and aVF.

Which of the following is the most appropriate treatment approach?

A

The patient should undergo primary percutaneous coronary intervention (PCI). He presents with evidence of ST-elevation myocardial infarction (STEMI). The goal of therapy in patients STEMI is to perform PCI within 90 minutes of presentation to a PCI-capable facility or within 120 minutes if the patient requires transfer from a non–PCI-capable hospital. However, patients may benefit from treatment up to 12 hours after the onset of symptoms and possibly even after this time, depending on clinical circumstances.

For patients with STEMI, including those presenting later than the optimal time frame, PCI is the preferred reperfusion strategy instead of treatment with thrombolytics. Thrombolytic therapy has not shown a clear benefit for patients with STEMI who present more than 12 hours after the onset of symptoms. It remains second-line treatment when PCI is not possible or is contraindicated.

Emergency coronary artery bypass graft surgery is not a routine method of revascularization in patients with STEMI. The reasons are logistical; it is nearly impossible to diagnose STEMI, perform cardiac catheterization, obtain access to an operating room, assemble a surgical team, and perform surgery within the necessary time frame to salvage the greatest amount of myocardium.

Compared with medical therapy alone, if performed promptly, coronary reperfusion (PCI or thrombolytic therapy) improves outcomes in nearly all groups of patients with acute STEMI.

How well did you know this?
1
Not at all
2
3
4
5
Perfectly
15
Q

A 70-year-old woman is hospitalized for an ST-elevation myocardial infarction involving the anterior wall. Her symptoms initially began 3 days before admission. The pain resolved spontaneously before she reached the hospital.

Two hours after presentation to the emergency department, she has acute onset of dyspnea and hypotension and requires emergent intubation. A portable chest radiograph shows cardiomegaly and pulmonary edema. Vasopressor therapy is initiated to support her blood pressure.

On physical examination, blood pressure is 90/60 mm Hg, pulse rate is 120/min, and respiration rate is 12/min. She has a grade 4/6 harsh holosystolic murmur at the right and left sternal borders associated with a palpable thrill. No S3 or S4 is heard. Crackles are heard bilaterally at the lung bases.

Which of the following is the most likely diagnosis?

A

A postinfarction ventricular septal defect (VSD) is the most likely cause of this patient’s symptoms. She initially presented with a delayed anterior wall ST-elevation myocardial infarction (STEMI). She then had acute respiratory distress and was found to have a new holosystolic murmur at the left sternal border on physical examination. These findings could be consistent with either a VSD or acute ischemic mitral regurgitation. However, given the location of the murmur and its association with a thrill, a VSD is more likely. With a postinfarction VSD, shunting of oxygenated blood from the left ventricle to the right ventricle occurs. This acute volume overload to the right ventricle results in cardiogenic shock and is rapidly fatal unless emergent surgical or possibly percutaneous intervention can be performed.

An acute aortic dissection associated with a myocardial infarction is more commonly associated with an inferior wall STEMI. Findings on physical examination may include asymmetric blood pressures and an early diastolic murmur of acute aortic insufficiency.

Rupture of the left ventricular free wall presents as hemopericardium with electromechanical dissociation and death. Risk factors include elderly age, female sex, first myocardial infarction, and anterior location of the infarction. The patient’s loud holosystolic murmur and thrill are not compatible with a free wall rupture.

Physical examination findings of right ventricular infarction include hypotension, clear lung fields, and elevated jugular venous pulsations. It would be exceedingly unusual for a right ventricular infarction to develop in association with an anterior wall infarction.

Acute ischemic mitral regurgitation is another late complication of myocardial infarction and also presents with a new holosystolic murmur. Ischemia with subsequent rupture of a papillary muscle or chordae tendineae is the most likely cause of acute ischemic mitral regurgitation and may result in a “flail” mitral valve. Echocardiography can accurately distinguish acute ischemic mitral regurgitation from a postinfarction VSD.

How well did you know this?
1
Not at all
2
3
4
5
Perfectly
16
Q

A 64-year-old man is evaluated in the emergency department because of chest pain. He describes the chest pain as nonradiating pressure in the midchest that began at rest 1 hour ago and is not associated with any symptoms. Medical history is remarkable for hypertension, type 2 diabetes mellitus, hyperlipidemia, and a 20-pack-year history of smoking. Medications are hydrochlorothiazide, metformin, and simvastatin.

On physical examination, he is afebrile, blood pressure is 140/80 mm Hg, pulse rate is 78/min, and respiration rate is 16/min. There is no jugular venous distention, the lungs are clear, and the findings on heart examination are normal.

Electrocardiogram shows a normal sinus rhythm and T-wave inversions in leads V2 through V6 without Q waves.

Initial cardiac biomarkers are within normal limits.

He is given aspirin, clopidogrel, low-molecular-weight heparin, and a nitrate, with resolution of his chest pain.

Which of the following is the most appropriate next step in management?

A

A β-blocker, such as metoprolol, should be added to this patient’s therapeutic regimen. He has acute-onset chest pain, normal cardiac biomarkers, and T-wave inversions shown on electrocardiogram. These findings are consistent with unstable angina. First-line therapies for patients with acute coronary syndromes, including unstable angina, include dual antiplatelet therapy with aspirin and a thienopyridine (e.g., clopidogrel, prasugrel, ticagrelor), a β-blocker, nitrates, and anticoagulation (e.g., heparin). Morphine may also be given in patients with active chest pain. These treatments attempt to minimize ischemia by addressing both the supply and demand of oxygen within myocardial cells.

When β-blockers are contraindicated for the treatment of unstable angina, as in a patient with severe bronchospastic lung disease that may be exacerbated by treatment, calcium channel blockers should be considered. However, the calcium channel blocker nifedipine causes an increase in heart rate and therefore myocardial oxygen demand and is contraindicated in patients with an acute coronary syndrome.

Reperfusion therapy, preferably with percutaneous coronary intervention or, alternatively, thrombolysis, is the mainstay of treatment for ST-elevation myocardial infarction. This patient has unstable angina based on his clinical presentation, findings on electrocardiogram, and normal cardiac biomarkers. Patients with unstable angina are usually treated initially with medical therapy, with further evaluation and treatment based on an assessment of the risk factors. Therefore, acute reperfusion therapy is not indicated in this patient.

How well did you know this?
1
Not at all
2
3
4
5
Perfectly
17
Q

A 54-year-old man is evaluated in the emergency department for an acute coronary syndrome that began 30 minutes ago. His medical history is significant for hypertension and type 2 diabetes mellitus. Medications are lisinopril and glipizide.

On physical examination, he is afebrile, blood pressure is 160/90 mm Hg, pulse rate is 80/min, and respiration rate is 12/min. Cardiovascular examination shows a normal S1 and S2 without an S3 and no murmurs. Lung fields are clear.

An initial serum troponin level is pending. Electrocardiogram shows 3-mm ST-segment elevation in leads V2 through V4 and a 1-mm ST-segment depression in leads II, III, and aVF.

Treatment is initiated with aspirin, clopridogrel, a β-blocker, and unfractionated heparin. His symptoms of chest pain improve.

Which of the following is the most appropriate next step in management?

A

This patient should undergo percutaneous coronary intervention (PCI). The electrocardiographic changes are consistent with acute anterior ST-elevation myocardial infarction (STEMI). Patients with STEMI who present within 12 hours of symptom onset should undergo either primary PCI or thrombolytic therapy. PCI is preferred because it is associated with a lower mortality rate compared with thrombolytic therapy. Ideally, it should be performed within 90 minutes of presentation to a facility with PCI capability or within 120 minutes if the patient requires transfer from a non–PCI-capable hospital. PCI also is indicated in patients with a contraindication to thrombolytic therapy and in patients with cardiogenic shock. PCI is most effective if completed within 12 hours of the onset of chest pain; the earlier the intervention, the better the outcome. Because this patient has no clear contraindication to PCI, this is the treatment of choice for STEMI.

Optimal management of patients with STEMI relies on timely recognition and rapid initiation of reperfusion therapy. Patients with an acute coronary syndrome and an electrocardiogram compatible with STEMI can be treated with reperfusion therapy without biomarker confirmation. Early biomarker results may be normal in patients with STEMI, and waiting for the results of the troponin level would delay appropriate treatment.

Medical therapy with further intervention based on risk stratification is indicated in patients with non–ST-elevation myocardial infarction or unstable angina, but would not be appropriate in this patient with STEMI.

How well did you know this?
1
Not at all
2
3
4
5
Perfectly
18
Q

A 77-year-old woman is admitted to the hospital for intermittent dizziness over the last few days. She reports no chest discomfort, dyspnea, palpitations, syncope, orthopnea, or edema. She underwent coronary artery bypass graft surgery 6 years ago after a myocardial infarction. She has hypertension and hyperlipidemia. Medications are hydrochlorothiazide, pravastatin, lisinopril, and aspirin.

On physical examination, blood pressure is 137/88 mm Hg and pulse rate is 52/min. The lungs are clear to auscultation. Cardiac auscultation shows bradycardia with regular S1 and S2 as well as an S4. A grade 2/6 early systolic murmur is heard at the left upper sternal border. Edema is not present.

On telemetry, she has sinus bradycardia with rates between 40/min and 50/min, with two symptomatic sinus pauses of 3 to 5 seconds each.

Which of the following is the most likely cause of this patient’s dizziness?

A

This patient has symptomatic sick sinus syndrome (SSS) because her episodes of sinus bradycardia are correlated with dizziness. SSS comprises a collection of pathologic findings that result in bradycardia. These include sinus arrest, sinus exit block, and sinus bradycardia. SSS is common in the elderly, and because symptoms can be intermittent or nonspecific, misdiagnosis can occur.

Because there is no dissociation between atrial depolarization and conduction to the ventricles on telemetry, the patient does not have complete heart block (third-degree heart block).

Complete heart block (third-degree heart block) is characterized by complete absence of conduction from the atria to the ventricles; the P waves and the QRS complexes are completely independent of each other. This patient’s P waves are followed by QRS complexes, and the pauses result from failure of the sinus node to fire (no P waves) and not from a failure of conduction from the atria to the ventricles.

The presence of P waves on telemetry excludes the diagnosis of atrial fibrillation, and the combination of conducted P waves and a narrow QRS complex makes nonsustained ventricular tachycardia unlikely.

How well did you know this?
1
Not at all
2
3
4
5
Perfectly
19
Q

A 73-year-old woman is evaluated during a routine examination. She has no symptoms and feels well. Her medications are levothyroxine for hypothyroidism and hydrochlorothiazide for hypertension. An electrocardiogram performed 2 years ago was normal.

On physical examination, heart rate is 42/min and regular. The remainder of the examination is normal. The thyroid-stimulating hormone level is normal. An electrocardiogram obtained as part of the current evaluation is shown :.

Which of the following diagnoses is confirmed by the electrocardiogram?

A

Third-degree atrioventricular block, or complete heart block, refers to a lack of atrioventricular conduction (characterized by lack of conduction of all atrial impulses to the ventricles), as seen in this patient’s electrocardiogram. This is an important diagnosis to make because pacemaker implantation may improve survival for patients with asymptomatic complete heart block; therefore, all patients with complete heart block should be treated with pacemaker implantation.

First-degree atrioventricular block is recognized electrocardiographically as a prolongation of the P-R interval; all P waves are conducted, and this condition requires no specific treatment. Second-degree atrioventricular block is characterized by intermittent nonconduction of P waves and subsequent “dropped” ventricular beats. Second-degree atrioventricular block is divided into two types, Mobitz type I and Mobitz type II. Mobitz type I second-degree atrioventricular block is characterized by progressive prolongation of the P-R interval until a dropped beat occurs. This type of heart block is characteristically transient and usually requires no specific treatment. Mobitz type II second-degree atrioventricular block is characterized by a regularly dropped beat (e.g., a nonconducted P wave every second or third beat), without progressive prolongation of the P-R interval. It is usually associated with evidence of additional disease in the conduction system, such as bundle branch block or bifascicular or trifascicular block. Mobitz type II atrioventricular block suddenly and unpredictably progresses to complete heart block and is usually treated with a pacemaker.

How well did you know this?
1
Not at all
2
3
4
5
Perfectly
20
Q

A 73-year-old man is evaluated in the emergency department for chest pressure with radiation to the arm, diaphoresis, and lightheadedness of 4 hours’ duration. His medical history includes a 20-year history of hypertension and type 2 diabetes mellitus. Medications are metformin, atenolol, and aspirin.

On physical examination, the patient is afebrile, blood pressure is 130/84 mm Hg in both arms, and pulse rate is 87/min and regular. The jugular vein is distended to 5 cm while the patient is upright. He has a faint left carotid bruit, bibasilar crackles to one-quarter up from the pulmonary bases, normal S1 and S2, and an S4 and an S3 with a grade 2/6 holosystolic murmur that is heard best at the apex to the axilla. Pulses are symmetric. The chest radiograph is consistent with heart failure. The electrocardiogram is shown :. An electrocardiogram obtained 6 months ago was normal. The initial serum troponin measurement is elevated.

Which of the following is the most likely diagnosis?

A

This patient is having an acute myocardial infarction manifesting as a new left bundle branch block on electrocardiogram and complicated by ischemic mitral regurgitation and heart failure. Electrocardiographically, left bundle branch block is associated with absent Q waves in leads I, aVL, and V6; a large, wide, and positive R wave in leads I, aVL, and V6; and prolongation of the QRS complex to greater than 0.12 seconds. Repolarization abnormalities are present and consist of ST-segment and T wave vectors directed opposite to the QRS complex. The presentation of acute coronary syndrome with new left bundle branch block should be considered equivalent to ST-elevation myocardial infarction and is an indication for acute reperfusion therapy. The diagnosis of acute aortic dissection should be considered in this patient, but the findings of symmetric blood pressures and pulse rates and normal mediastinum on chest radiograph are reassuring. Aortic dissection chest pain is often described as ripping and tends to radiate to the back. Although chest pain, left bundle branch block, mitral regurgitation, and heart failure may all be complications of endocarditis, the acute onset of this patient’s symptoms and the fact that he is afebrile make this diagnosis much less likely than acute myocardial infarction. Pericarditis also generally has a more subacute presentation, and typical electrocardiographic findings include diffuse PR depressions or ST-segment elevations and not new left bundle branch block.

How well did you know this?
1
Not at all
2
3
4
5
Perfectly
21
Q

A 67-year-old man is evaluated because of a 10-day history of lightheadedness and increasing frequency of chest pain and palpitations for the last 3 days. He denies syncope. The medical history is significant for coronary artery disease and difficult-to-control hypertension. Medications are aspirin, nitroglycerin, atenolol, hydrochlorothiazide, and sustained release diltiazem.

On physical examination, blood pressure is 156/72 mm Hg; pulse rate is 60/min, with occasional irregularity; and respiration rate is 16/min. Jugular venous pressure is normal. The lungs are clear to auscultation. Cardiac examination shows a physiologically split S2, an S4, and no S3. There is a grade 2/6 systolic murmur that is best heard in the second right intercostal space. The remainder of the physical examination is unremarkable.

The patient’s electrocardiogram is shown :.

Which of the following is the electrocardiographic diagnosis?

A

This patient has second-degree atrioventricular block, diagnosed by the presence of isolated P waves that are not followed by a QRS complex. Second-degree atrioventricular block is divided into two types, Mobitz type I and Mobitz type II. This patient has second-degree atrioventricular block, Mobitz type I (Wenckebach), with progressive prolongation of the PR interval until the “dropped” beat. Additionally, in a Mobitz type I rhythm, the PP interval is constant but the PR interval decreases progressively until the dropped beat occurs. This patient’s rhythm is most likely caused by the combination of atenolol and diltiazem, both of which can decrease conduction within the atrioventricular node. Mobitz type II atrioventricular block is much less common. It is usually associated with a right or left bundle branch block pattern, and the dropped beat is associated with an intermittent block of the remaining bundle branch, which indicates significant conduction system disease. A pacemaker is commonly required. In Mobitz type II atrioventricular block, the PR interval is constant in the conducted beats and the RR interval containing the nonconducted (dropped) beat is equal to two PP intervals.

First-degree atrioventricular block is characterized by a prolonged PR interval of more than 0.2 seconds. Complete heart block (third-degree atrioventricular block) is characterized by complete absence of conduction from the atria to the ventricles; the P waves and the QRS complexes are completely independent of each other. Careful analysis shows that the P wave rate and the QRS complex rate are different and that the PR interval is different for every QRS complex.

How well did you know this?
1
Not at all
2
3
4
5
Perfectly
22
Q

A 32-year-old woman is evaluated in the emergency department because of intermittent palpitations and dizziness for the last week. She has not experienced chest pain, dyspnea, or orthopnea. She was ill 6 weeks ago with fever, fatigue and myalgias, and an associated erythematous rash on her abdomen that resolved over 2 weeks. She has no significant medical history. She works as a landscaper.

On physical examination, temperature is normal, blood pressure is 120/70 mm Hg, and pulse rate is 45/min. The cardiac examination shows bradycardia, but findings are otherwise unremarkable. The remainder of the physical examination is normal.

An electrocardiogram shows sinus rhythm with a heart rate of 90/min, with complete heart block and a junctional escape rate of 50/min.

In addition to hospitalization for cardiac monitoring, which of the following is the most appropriate management for this patient?

A

This patient should be treated with intravenous ceftriaxone. She most likely has Lyme carditis manifested by acute-onset, high-grade atrioventricular conduction defects that occasionally may be associated with myocarditis. Carditis occurs in 5% to 10% of patients with Lyme disease, usually within a few weeks to months after infection. Atrioventricular block can present in any degree, and progression to complete heart block is often rapid. The prognosis is good, usually with resolution of atrioventricular block within days to weeks. The presence of the characteristic skin rash, erythema migrans, with or without a history of tick bite in an endemic region, has a greater than 80% probability of being caused by Borrelia burgdorferi infection and is sufficient to support a decision to treat Lyme disease empirically without laboratory confirmation of the diagnosis. The preferred antibiotic regimen is intravenous ceftriaxone until the heart block resolves, followed by a 21-day course of oral therapy.

An electrophysiology study is not indicated because it would not provide additional prognostic information. Atrioventricular block is usually within the node, but sinoatrial and His-Purkinje involvement has also been described.

Most cases of Lyme carditis resolve spontaneously, and neither temporary nor permanent pacemaker therapy is needed. Temporary pacing would be required if the patient were hemodynamically unstable with bradycardia. However, this rarely occurs in the setting of Lyme carditis. Indications for permanent pacemaker placement would include persistent high-grade atrioventricular block.

How well did you know this?
1
Not at all
2
3
4
5
Perfectly
23
Q

A 62-year-old man is evaluated in the emergency department for a 2-day history of dyspnea and palpitations. His other medical problems include a 10-year history of hypertension and diabetes mellitus. Medications are lisinopril and metformin.

On physical examination, temperature is 37.0°C (98.6°F), blood pressure is 130/86 mm Hg, pulse rate is 132/min, respiration rate is 18/min, and oxygen saturation is 94% on ambient air. Cardiac examination shows a rapid heart rate but no other abnormalities.

Electrocardiogram is shown :.

Which of the following is the most appropriate treatment?

A

The most appropriate treatment for the patient is diltiazem given intravenously. He has atrial fibrillation with a rapid ventricular response. Atrial fibrillation is caused by rapid and uncoordinated electrical activation within the atria. The electrocardiogram shows an absence of P waves and an irregular ventricular response. Approximately 50% of episodes of atrial fibrillation convert to a normal rhythm spontaneously.

If the patient is symptomatic but hemodynamically stable, the initial goal is to reduce the heart rate to 60/min to 110/min with a rate control agent. Intravenous options include diltiazem, verapamil, metoprolol, and esmolol. Digoxin can be used as a second agent, especially in patients with heart failure or systolic dysfunction, but peak effect can take up to 6 hours. In those with rapid atrial fibrillation with minimal or mild symptoms, oral agents may be used.

Adenosine is an intravenous agent that transiently blocks atrioventricular nodal conduction; it is effective at terminating supraventricular tachycardia where the electrical circuit is dependent on the atrioventricular node. Intravenous adenosine may be used to break the reentrant rhythm in atrioventricular nodal reentrant tachycardia, or orthodromic atrioventricular reciprocating tachycardia. However, it is not an effective therapy for atrial fibrillation.

Patients with adverse or marginal hemodynamic status or acute coronary ischemia because of a rapid ventricular response should undergo immediate cardioversion. Cardioversion is also the treatment of choice for patients with wide complex tachycardia. This patient is hemodynamically stable, and immediate electrical cardioversion is not necessary.

How well did you know this?
1
Not at all
2
3
4
5
Perfectly
24
Q

A 69-year-old man is evaluated in the emergency department for atrial fibrillation with a rapid ventricular response. His heart rate is controlled with intravenous diltiazem. Other medical problems include a myocardial infarction 5 years ago and hypertension. The patient had a transient ischemic attack 5 months ago. Medications are metoprolol, lisinopril, simvastatin, and aspirin.

On physical examination, the patient is afebrile, blood pressure is 165/90 mm Hg, pulse rate is 88/min and regular, respiratory rate is 20/min, and oxygen saturation is 97% on ambient air. Cardiac examination shows an irregular rhythm. There is no jugular venous distention or peripheral edema, and the lungs are clear to auscultation.

Echocardiogram shows a left ventricular ejection fraction of 55% and moderate dilation of the left atrium.

Which of the following should be included in the initial management of this patient’s atrial fibrillation?

A

The patient should receive long-term warfarin therapy to prevent cardiogenic thromboembolism related to the atrial fibrillation. In patients without significant valvular disease, the most commonly used method to determine the choice of thromboprophylaxis is the CHADS2 score. One point each is given for congestive heart failure, hypertension (even if treated), age 75 years or older, and diabetes; 2 points are given for previous stroke or transient ischemic attack. Patients with zero points have a low stroke risk, and aspirin or no treatment is used. Those with a score of 1 have an intermediate stroke risk, and either warfarin or aspirin is used, based on physician and patient preference. A score of 2 or more on this scale indicates high risk, and therapeutic anticoagulation is preferred. Recently, the CHADS2 score was updated to the CHA2DS2-VASc score, which better recognizes the influence of gender and the presence of established vascular disease as stroke risk factors. The CHA2DS2-VASc score also weights age more heavily as a risk factor. For a CHA2DS2-VASc score of 1, either anticoagulation or antiplatelet therapy is indicated. For a CHA2DS2-VASc score of 2 or greater, oral anticoagulation is recommended for prevention of stroke. This patient has a CHADS2 score of 3 points (hypertension and transient ischemic attack), and a CHA2DS2-VASc score of 4 points (age range, gender, hypertension, and transient ischemic attack). Using either scoring system, therapeutic anticoagulation, such as with warfarin, is indicated.

A new class of anticoagulation agents is the direct thrombin inhibitors. These drugs, such as dabigatran, have been shown to be slightly superior to warfarin in preventing ischemic and hemorrhagic stroke, although their anticoagulant effect is not easily reversed and they are substantially more expensive than warfarin. These agents may be an alternative to warfarin in patients with nonvalvular atrial fibrillation.

In patients who are not candidates for warfarin treatment, adding clopidogrel to aspirin can reduce the risk of stroke more than aspirin alone, although the risk of major bleeding events is increased. In patients who are able to take warfarin, however, clopidogrel plus aspirin is inferior for preventing ischemic stroke.

Atrial fibrillation ablation is an option in patients who are symptomatic and have been treated unsuccessfully with at least one antiarrhythmic agent. The procedure entails electrical isolation of the pulmonary veins so that premature atrial contractions, which frequently originate in the pulmonary veins, cannot initiate atrial fibrillation. Atrioventricular node ablation is another option for patients who do not benefit from medical therapy. A pacemaker is inserted at the time of ablation, so the ventricular rate is controlled by the device. The patient does not meet the criteria for either of these procedures.

25
Q

A 36-year-old woman is evaluated in the emergency department for an episode of chest pain, palpitations, and lightheadedness. The symptoms persisted for 1 hour and ceased shortly after she arrived at the hospital. She has no history of cardiac disease, but has a 6-month history of hypertension that is controlled with chlorthalidone 25 mg daily. She has occasional brief episodes of rapid palpitations provoked by exertion. She does not have exercise intolerance or exertional chest discomfort.

On physical examination, blood pressure is 154/94 mm Hg, without orthostatic change. Pulse rate is 60/min and regular. Findings on physical examination are remarkable only for a paradoxically split S2. The patient’s electrocardiogram is shown :.

Which of the following best describes the electrocardiographic finding for this patient?

A

The electrocardiogram is diagnostic for ventricular preexcitation by an accessory atrioventricular connection, with a short P-R interval, prolonged QRS duration, and slurred onset of the QRS (delta wave) interval. With paroxysmal tachyarrhythmias, this pattern is diagnostic for Wolff-Parkinson-White syndrome and accounts for the paradoxically split S2. In a symptomatic patient with recurrent palpitations or syncope, radiofrequency ablation of the accessory pathway is likely to provide the most definitive improvement.

The electrocardiogram does not show atrial fibrillation because it shows clearly discernible P waves and a regular rhythm. First-degree atrioventricular block is associated with a P-R interval of more than 0.20 seconds, not a shortened interval (<0.11 seconds), as seen in this case.

26
Q

A 71-year-old man is evaluated in the clinic for symptoms of palpitations, shortness of breath, and decreased exercise tolerance for the last 2 days. His medical history is significant for hypertension. His only medication is lisinopril.

On physical examination, blood pressure is 115/62 mm Hg and pulse rate is 152/min. The lungs are clear. Cardiac examination is significant for tachycardia, but no extra heart sounds or murmurs are noted.

The electrocardiogram is shown :.

Which of the following is the most likely cause of this patient’s symptoms?

A

The electrocardiogram shows atrial flutter, which is the likely cause of the patient’s clinical symptoms. Atrial flutter is a rapid, regular atrial rhythm that appears as a “saw-tooth” pattern on an electrocardiogram, often best seen in the inferior leads. The atrial rate is characteristically approximately 300/min, with atrioventricular conduction occurring once every two flutter waves (2:1 atrioventricular conduction), a very typical pattern for atrial flutter.

Atrial fibrillation is more disorganized atrial activity at a rate of 350 to 600/min, with no discernible P waves. It is characteristically associated with an irregularly irregular rhythm, and the fibrillatory waves vary in amplitude, morphologic pattern, and interval, creating a rough, irregular baseline between QRS complexes. Multifocal atrial tachycardia is defined by the electrocardiographic finding of discrete P waves with at least three morphologic patterns with varying P-P, P-R, and R-R intervals. The morphologic features of P waves are generally best seen in leads II, III, and V1. In adults, multifocal atrial tachycardia is often associated with other disorders, often hypoxic chronic obstructive pulmonary disease or electrolyte abnormalities. Sinus tachycardia is a sinus rhythm with a ventricular rate greater than 100/min. The P waves have normal morphologic features but can become difficult to see with heart rates greater than 140/min because the P waves begin to merge with the preceding T waves. Slowing the heart rate with carotid sinus massage often shows the hidden P waves and establishes the diagnosis.

27
Q

A 68-year-old man is evaluated in the emergency department for shortness of breath and palpitations that have worsened over the last 3 days. His medical history is significant for chronic obstructive pulmonary disease. He has a 50-pack-year smoking history and continues to smoke 1 pack of cigarettes daily. His medications include tiotropium and albuterol metered-dose inhalers.

On physical examination, temperature is 37.8°C (100°F) and pulse rate is 122/min. Oxygen saturation on ambient air is 89%. The patient is in moderate respiratory distress. Chest examination shows decreased airflow with diffuse expiratory wheezing. Cardiac examination shows distant heart sounds and an irregular rate with a loud S2 and no murmurs.

Laboratory studies show normal electrolytes. The electrocardiogram is shown :.

Which of the following is the most likely electrocardiographic diagnosis?

A

The electrocardiogram shows multifocal atrial tachycardia (MAT). MAT is most commonly seen in acutely ill patients, most often in the setting of pulmonary disease (such as the exacerbation of chronic obstructive pulmonary disease with associated hypoxia in this patient) and electrolyte abnormalities. The electrocardiogram in MAT is defined by the presence of discrete P waves with at least three different morphologic patterns with varying P-P, P-R, and R-R intervals. The morphologic features of P waves are generally best seen in leads II, III, and V1. Therapy is directed at the underlying disease process because without treatment the arrhythmia is often refractory to therapy or may recur. In this patient, treatment should be directed toward the pulmonary disease and correction of electrolyte imbalances, especially magnesium. Therefore, the treatment of choice in this patient is oxygen, inhaled bronchodilators, antibiotics, and corticosteroids. Electrolyte levels should be corrected, and the heart rate may be controlled as needed by atrioventricular blocking agents.

Atrial fibrillation is characterized by disordered atrial electrical activity that leads to an irregular heart rate similar to MAT; however, P waves are not detectable, as in this patient’s electrocardiogram. In atrioventricular nodal reentrant tachycardia, the atria and ventricles are activated simultaneously from a reentrant circuit within the atrioventricular node. The QRS complex is narrow, and no P waves are seen. Atrioventricular reciprocating tachycardia is a bypass tract–mediated reentrant tachycardia in which the anterograde conduction (atrium-to-ventricle) is typically via the atrioventricular node and retrograde conduction is via the bypass tract. Because bypass tract conduction is typically faster than conduction via the atrioventricular node, atrial activation occurs rapidly after the QRS complex, resulting in a “short R-P” tachycardia, and the P wave is usually located within the ST segment. Both atrioventricular reciprocating tachycardia and atrioventricular nodal reentrant tachycardia are regular rhythms.

28
Q

A 34-year-old woman is evaluated in the emergency department after the acute onset of palpitations approximately 1 hour ago. She reports no shortness of breath, chest pain, presyncope, or syncope. The medical history is unremarkable, and there is no family history of sudden cardiac death. She takes no medications.

On physical examination, she is afebrile, blood pressure is 118/64 mm Hg, and pulse rate is 165/min. Other than a regular, rapid pulse, the cardiopulmonary examination is normal.

Baseline electrocardiogram shows a narrow complex tachycardia at 165/min. Adenosine is given as a rapid intravenous push, and the patient converts to normal sinus rhythm.

Which of the following is the most likely diagnosis?

A

The patient’s arrhythmia is most likely atrioventricular nodal reentrant tachycardia (AVNRT). Supraventricular tachycardias (SVTs) are rapid heart rhythms that require atrial tissue or the atrioventricular node for initiation and maintenance. Although atrial fibrillation and atrial flutter are technically SVTs, the term is generally used to describe the most frequent paroxysmal SVTs: AVNRT, atrioventricular reciprocating tachycardia, and atrial tachycardia. The most common paroxysmal SVT is AVNRT, which involves a slow pathway and a fast pathway within the atrioventricular node. The slow pathway conducts slowly but repolarizes quickly, whereas the fast pathway conducts quickly but repolarizes slowly. Typical AVNRT (slow-fast) often has an R-P interval so short that the P wave is buried within the QRS complex. The QRS complex is usually narrow as long as conduction below the atrioventricular node is normal.

Adenosine, if pushed rapidly through an intravenous line, can be used to diagnose and treat supraventricular tachycardias. Up to 95% of reentrant atrioventricular node-dependent tachycardias, such as AVNRT and AVRT, terminate with adenosine if administered properly. Adenosine transiently blocks AV nodal conduction, interrupting the reentrant circuit and terminating the AV node-dependent arrhythmia.

Adenosine given in the presence of other supraventricular arrhythmias, including atrial tachycardia, atrial fibrillation, and sinus tachycardia, should only slow the ventricular rate; this may be useful in diagnosis because it may show the underlying rhythm.

29
Q

A 19-year-old man is evaluated after the recent sudden death of his father at 45 years of age. He reports no chest pain, shortness of breath, palpitations, dizziness, or syncope. He does not smoke or use drugs and is not hypertensive. He takes no medication. The patient has no siblings.

On physical examination, he is afebrile, blood pressure is 120/60 mm Hg, pulse rate is 60/min, and respiration rate is 14/min. A grade 2/6 midsystolic murmur that increases during the strain phase of the Valsalva maneuver is heard. The lungs are clear to auscultation.

Electrocardiogram shows sinus rhythm and increased QRS voltage in the precordial leads. Echocardiogram shows asymmetric basal and midseptal hypertrophy, a thickened septum, and increased left ventricular outflow tract gradient.

Which of the following is the most appropriate management?

A

This patient should undergo placement of an implantable cardioverter-defibrillator (ICD). Echocardiography is virtually diagnostic for hypertrophic cardiomyopathy (HCM). His current risk stratification shows two major risk factors for sudden cardiac death: family history of premature sudden death in a first-degree relative and significant left ventricular wall thickening. Management in this asymptomatic patient is predominantly focused on prevention of sudden cardiac death. ICD implantation is effective for primary prevention of sudden cardiac death in patients with HCM.

In patients with outflow tract obstruction and symptoms of heart failure that are refractory to medication, alleviation of the obstruction with septal myectomy or alcohol septal ablation may be indicated. The patient is asymptomatic; therefore, alcohol septal ablation is not indicated.

Electrophysiology testing is not a reliable method to stratify risk for sudden cardiac death in patients with HCM and is not an appropriate test for this patient.

Amiodarone may be used for primary prevention of sudden cardiac death in patients with HCM with one or more major risk factors, but its efficacy is not established. It may be considered when ICD implantation is not feasible.

American Heart Association consensus recommendations on physical activity in patients with HCM note that most low-level recreational activities, such as bowling or golf, are probably permissible. High-level recreational activities, such as basketball or bodybuilding, are not advised or are strongly discouraged. Recommendations on moderate-level recreational activities vary. Some activities, such as tennis, are deemed probably permissible, whereas others, including weightlifting, are strongly discouraged. Patients with HCM require formal counseling about acceptable levels of physical activity.

30
Q

A 54-year-old man is evaluated in the emergency department because of a 2-hour history of palpitations. He reports no syncope, presyncope, chest pain, or shortness of breath. He has had no previous episodes of palpitations. Medical history is significant for nonischemic cardiomyopathy. The ejection fraction was most recently measured at 38%. Medications are carvedilol and candesartan.

On physical examination, the patient is afebrile, blood pressure is 125/86 mm Hg, and pulse rate is 110/min. Cardiac evaluation shows a regular rate and rhythm. The remainder of the findings on physical examination are unremarkable.

The electrocardiogram is shown :.

A

The most appropriate treatment for this patient is amiodarone. The electrocardiogram shows : a regular, monomorphic wide-complex tachycardia. Although it is possible that the patient has supraventricular tachycardia with aberrancy or antidromic atrioventricular reciprocating tachycardia, in a patient with wide-complex tachycardia and a history of coronary artery disease or cardiomyopathy, ventricular tachycardia is the assumed diagnosis. Hemodynamic stability does not exclude a diagnosis of ventricular tachycardia. The first-line treatment for a hemodynamically stable ventricular tachycardia is an intravenous antiarrhythmic agent such as amiodarone. Procainamide and sotalol are also acceptable agents, and lidocaine can be used as a second-line agent.

Immediate cardioversion is not necessary because this patient does not have signs of instability. If an antiarrhythmic agent is not successful, then elective cardioversion with sedation can restore normal rhythm.

Adenosine may be given for a stable wide-complex rhythm to determine whether it is a supraventricular tachycardia with aberrant conduction or an antidromic atrioventricular reciprocating tachycardia. However, because of the patient’s high risk of ventricular tachycardia given his known coronary artery disease, this would not be the most appropriate initial intervention.

The administration of verapamil or β-blockers is not indicated in patients with stable ventricular tachycardia. These drugs can cause severe hemodynamic deterioration and lead to ventricular fibrillation and cardiac arrest.

This patient should be offered an implanted cardioverter-defibrillator for long-term prevention of sudden cardiac death.

31
Q

A 47-year-old woman is evaluated for a 4-month history of a sensation of “thumping” in her chest. She reports feeling as if her heart stops when these episodes occur. The symptoms occur frequently throughout the day but are more noticeable at night. She finds them bothersome and notes that her symptoms appear to decrease with exercise. She reports no chest pain, dyspnea, orthopnea, or edema. She is healthy and active and takes no medications.

On physical examination, the patient is afebrile, blood pressure is 110/67 mm Hg, and pulse is 72/min with occasional ectopy. Cardiac auscultation is normal except for occasional extra beats. There are no murmurs, gallops, or clicks. The remainder of her examination is unremarkable.

Electrocardiogram shows sinus rhythm with normal intervals and occasional premature ventricular contractions with varying morphologic patterns that correspond to her symptoms of palpitations. Echocardiogram shows a structurally normal heart.

Which of the following is the most appropriate treatment for this patient?

A

This patient has frequent symptomatic premature ventricular complexes. Premature ventricular complexes are spontaneous depolarizations originating from the ventricles. A premature complex may be single, may occur in pairs (couplets), or may alternate with sinus beats in a specific multiple, such as in bigeminy, in which premature ventricular complexes and sinus beats alternate in a 1:1 ratio. These beats are usually followed by a compensatory pause, and patients may feel as if their heart is stopping. In the absence of structural heart disease, the prognosis is benign. Treatment is based on ameliorating significant symptoms, which may include palpitations, fatigue, and lightheadedness. If the patient’s symptoms are tolerable, no therapy is indicated. If not, β-blockers, such as metoprolol, are reasonably effective at suppressing premature ventricular complexes. In contrast, in patients with heart failure, hypertension, or left ventricular hypertrophy, ventricular ectopy has been described as a marker of increased risk of cardiovascular events.

The high incidence of side effects and organ system toxicity with amiodarone therapy proscribes its use as a first-line agent for suppression of premature ventricular complexes.

A pacemaker generally has no primary role in the treatment of symptomatic premature ventricular complexes.

Radiofrequency ablation is usually reserved for patients with severe symptoms that are refractory to drug therapy and those with more sustained ventricular arrhythmias, particularly those with ectopy originating from the right ventricular outflow tract. In the current patient, the multifocal nature of the patient’s ectopy, as documented by the varying morphologic patterns of the premature ventricular complexes, would reduce the efficacy of the procedure.

32
Q

A 19-year-old man is evaluated in the emergency department because of syncope. He was exercising when he lost consciousness. He had no previous symptoms. He recovered quickly and had no residual symptoms. He had three similar episodes in the past but did not seek evaluation. He did not consider these episodes significantly abnormal because his mother has similar symptoms periodically. He is physically active and has no other symptoms. He takes no medications. The medical history is otherwise unremarkable.

On physical examination, the patient is afebrile, blood pressure is 122/75 mm Hg, pulse rate is 67/min and regular, and respiration rate is 12/min. Findings of cardiac examination are normal, and the remainder of the physical examination is unremarkable.

An electrocardiogram is ordered.

Which of the following electrocardiographic findings is most likely to provide a diagnosis?

A

This patient most likely has long QT syndrome (LQTS). Cardiac events in patients with LQTS include syncope and cardiac arrest as a result of torsade de pointes ventricular tachycardia. LQTS may be either congenital or acquired. This patient likely has congenital LQTS, suggested by recurrent syncope triggered by activity and a family history of similar symptoms. This diagnosis should be considered in patients who have family members with sudden death. Risk factors for acquired LQTS include female sex, hypokalemia, hypomagnesemia, structural heart disease, previous QT-interval prolongation, and a history of drug-induced arrhythmia. Other cardiac causes of syncope and sudden death in young patients include hypertrophic cardiomyopathy and arrhythmogenic right ventricular dysplasia.

Left bundle branch block (LBBB) and right bundle branch block (RBBB) are electrocardiographic patterns that increase in frequency with age. LBBB most often occurs in patients with underlying heart disease. In older patients, LBBB is associated with increased mortality. In younger patients, however, LBBB is not associated with syncope or sudden death and the prognosis is generally excellent. RBBB is similarly associated with increased mortality in older patients with underlying heart disease. When RBBB is not associated with underlying cardiac disease, patient outcomes are generally excellent. RBBB is an unlikely cause of this patient’s symptoms.

First-degree atrioventricular block is characterized by prolongation of the P-R interval to more than 0.2 seconds. It usually is not associated with alterations in heart rate and has no association with syncope or sudden death.

33
Q

A 64-year-old man was admitted to the hospital 2 days ago with an acute ST-elevation myocardial infarction. At the time of presentation he became unresponsive because of ventricular fibrillation. He was resuscitated successfully and underwent stenting of a complete occlusion of his proximal left anterior descending artery in the cardiac catheterization laboratory. He has done well since that time. Current medications are aspirin, clopridogrel, metoprolol, lisinopril, and atorvastatin.

On physical examination, the patient is afebrile, blood pressure is 115/72 mm Hg, pulse rate is 65/min, and respiration rate is 12/min. There is no jugular venous distention, the lungs are clear, and findings of heart examination are unremarkable.

Electrocardiogram shows a resolving anterior myocardial infarction but is otherwise unremarkable. Echocardiogram shows mild hypokinesis of the anterior wall and left ventricular ejection fraction of 45%.

A

The best option at this time is continued medical management. Ventricular tachyarrhythmias are common in the setting of acute myocardial infarction, occurring in up to 20% of patients. Despite a sixfold increase in the in-hospital mortality rate, the overall mortality rate at 1 year is not increased in patients with ventricular fibrillation that occurs early in this setting. Therefore, unlike sudden cardiac death occurring in other settings, cardiac arrest within the first 48 hours of transmural acute myocardial infarction does not require cardioverter-defibrillator placement.

Primary ventricular fibrillation should be distinguished from ventricular fibrillation that occurs later in the course of disease, usually as a result of heart failure. Before the advent of the implantable cardioverter-defibrillator, ventricular fibrillation occurring late in the hospital course was associated with a 1-year mortality rate of 85%. All patients, even those who have not had arrhythmia during myocardial infarction, should be reevaluated after myocardial infarction with echocardiography to further stratify risk. If the ejection fraction is reduced (<35%), the patient may be a candidate for cardioverter-defibrillator placement.

Amiodarone has not been shown to improve the overall mortality rate after myocardial infarction. In the general population of patients who have survived cardiac arrest, amiodarone does not improve the mortality rate.

Implantable cardioverter-defibrillator placement is not indicated for patients who experience ventricular arrhythmias less than 48 hours after an acute ST-elevation myocardial infarction. Implantable cardioverter-defibrillators have shown a mortality benefit for essentially all other groups of patients who have survived cardiac arrest.

Typical indications for pacemaker placement include symptomatic sinoatrial node dysfunction (sinus bradycardia, intra-atrial block, exit block) and symptomatic bradycardia as a result of advanced second- or third-degree heart block. The current patient, who is asymptomatic and has no evidence of bradycardia or advanced heart block on electrocardiogram, has no indication for pacemaker placement. Pacemaker placement does not prevent sudden death as a result of ventricular tachyarrhythmias.

34
Q

A 68-year-old man is evaluated during a routine appointment for stable heart failure. He has a 1-year history of New York Heart Association class III heart failure as a result of hypertensive cardiomyopathy. He has symptoms with minimal exertion, such as walking one block or climbing half a flight of stairs. Medications are furosemide, lisinopril, digoxin, and metoprolol. An implantable cardioverter-defibrillator was placed 6 months ago, after an echocardiogram showed left ventricular ejection fraction of 30%.

On physical examination, the patient is afebrile, blood pressure is 130/78 mm Hg, pulse rate is 78/min, respiration rate is 18/min, and oxygen saturation is 96% on ambient air. Cardiac examination shows a summation gallop but no murmurs. Estimated central venous pressure is 10 cm H2O. Bilateral lower lobe crackles are present. There is trace ankle edema.

The serum creatinine level is 1.2 mg/dL (106.08 µmol/L), and the serum potassium level is 4.5 meq/L (4.50 mmol/L).

Which of the following is the most appropriate addition to this patient’s heart failure medications?

A

The most appropriate addition to this patient’s medications is spironolactone. Indications for specific medications for systolic heart failure are generally based on the patient’s functional status as measured by New York Heart Association (NYHA) functional class. In the absence of contraindications or intolerance, treatment with an angiotensin converting enzyme (ACE) inhibitor (eg, lisinopril) and a β-blocker (eg, metoprolol) is indicated for all patients with systolic heart failure, regardless of functional status or symptom status. The addition of spironolactone to ACE inhibitor and β-blocker therapy is indicated for patients with severe systolic heart failure (NYHA class III-IV) and is associated with a 30% reduction in mortality rate (including sudden death and death as a result of progressive heart failure), reduction in hospitalizations, and improved NYHA functional class. Care should be taken to prescribe spironolactone according to evidence-based guidelines (NYHA class III-IV symptoms, serum potassium level <5 meq/L, and creatinine level <2.5 mg/dL), and should include close clinical and laboratory follow-up, with particular attention to serum potassium levels.

Routine combination of ACE inhibitor and angiotensin receptor blocker (ARB) therapy (eg, valsartan) is not recommended because of the increased risk of adverse effects, including acute kidney injury, hyperkalemia, and hypotension, without proven clinical benefit. Calcium channel blockers are not preferred agents for the treatment of systolic heart failure. These agents are not associated with improved outcomes and have been implicated in heart failure decompensation (13% in one study). These results are likely related to negative inotropic effects, particularly with the older-generation agents such as nifedipine. Amlodipine and felodipine are the only two calcium channel blockers that have been tested in large clinical trials to have neutral effects on mortality rates in patients with systolic heart failure. These agents would be acceptable to use for the treatment of conditions such as hypertension or angina that are not adequately controlled with the other evidence-based medications. They are not indicated for the treatment of systolic heart failure itself.

35
Q

A 70-year-old woman is evaluated in the emergency department because of dyspnea. She has had progressive shortness of breath over the last 5 days associated with a nonproductive cough. Symptoms are greatest with exertion and possibly when lying down. She has no fever, chest pain, or increase in her chronic lower extremity edema. Her other medical problems are chronic venous insufficiency, chronic obstructive pulmonary disease, hypertension, and chronic kidney disease. Medications are chlorthalidone, amlodipine, lisinopril, and tiotropium inhaler.

On physical examination, she is afebrile, blood pressure is 144/88 mm Hg, pulse rate is 90/min, respiration rate is 22/min, and oxygen saturation is 94% on ambient air. Body mass index is 30. Because her neck is large, it is not possible to estimate central venous pressure. Breath sounds are distant, with occasional end-expiratory wheezing. Heart sounds are distant, and extra sounds or murmurs are not detected. There is 2+ ankle edema with hyperpigmentation localized to the medial aspect of the ankles.

Frontal chest radiograph shows increased radiolucency of the lung, flat diaphragms, and a narrow heart shadow. Electrocardiogram shows evidence of left ventricular hypertrophy. Laboratory tests, including serum creatinine, electrolytes, and B-type natriuretic peptide levels, are pending.

Which of the following factors increases the risk of a falsely low B-type natriuretic peptide level?

A

Obesity increases the risk of a falsely low B-type natriuretic peptide (BNP) level. BNP, or N-terminal proBNP, levels should be assessed in patients suspected of having heart failure, particularly when the diagnosis or relative contribution of heart failure to symptoms is uncertain. BNP is especially helpful in differentiating dyspnea as a result of heart failure versus dyspnea as a result of pulmonary disease. Among patients presenting to the emergency department with dyspnea of undetermined cause, a BNP level of less than 100 pg/mL accurately excludes decompensated heart failure as a cause. However, among ambulatory patients with established heart failure, “normal” ranges for BNP during periods of clinical stability may be as high as 500 pg/mL. Factors other than heart failure that affect BNP levels include kidney failure, older age, and female sex, all of which increase BNP level. The BNP level is also elevated in pulmonary embolism, acute myocardial infarction, and acute tachycardia, which are conditions that increase ventricular strain. Obesity reduces BNP levels. Interpretation of BNP results should take these factors into account.

36
Q

A 55-year-old man is evaluated because of a 2-month history of dyspnea on exertion without chest pain. Medical history is significant for type 2 diabetes mellitus, hypertension, and hyperlipidemia. He has a 30-pack-year smoking history but does not currently use tobacco products. Medications are metformin, lisinopril, pravastatin, and aspirin.

On physical examination, blood pressure is 110/75 mm Hg and pulse rate is 60/min. Jugular venous distention is noted, and trace lower extremity edema is present. The point of maximal impulse is normal in size and location. Cardiac examination shows regular rate and rhythm, and the chest is clear to auscultation.

Electrocardiogram is shown :. Echocardiogram shows inferior wall hypokinesis and ejection fraction of 35%.

Which of the following is the most appropriate diagnostic test to perform next?

A

Coronary angiography is indicated to evaluate coronary artery disease (CAD) as a cause of newly diagnosed left ventricular systolic dysfunction. The patient has diabetes mellitus, which is a risk factor for CAD, and evidence of a previous inferior wall myocardial infarction on electrocardiogram (Q waves in leads II, III, and aVF). Indications for coronary angiography for evaluation of new-onset heart failure include angina or new-onset left ventricular dysfunction in the setting of a condition, such as diabetes, that may predispose to silent ischemia. Because revascularization may improve left ventricular function, some experts recommend a low threshold for evaluation for CAD.

Noninvasive stress testing, such as with an adenosine thallium test or a dobutamine echocardiographic stress test, would likely suggest CAD in this patient. However, given the relatively high pretest probability, a noninvasive stress test would not change that probability. The more relevant issue is whether the coronary anatomy is amenable to revascularization, which could improve function. Therefore, coronary angiography is the most appropriate diagnostic test.

Cardiac magnetic resonance imaging is useful for evaluating possible infiltrative or inflammatory cardiomyopathy. Given the significant evidence of CAD in this patient, the likelihood of an infiltrative or inflammatory cardiomyopathy is very low, and cardiac magnetic resonance imaging would not be an appropriate first diagnostic test.

Although this patient has some limiting symptoms (dyspnea on exertion) and a history of smoking, he has clinical evidence of heart failure. Pulmonary function testing would not aid in assessing the cause of new-onset heart failure nor would it be useful in diagnosing potential CAD.

37
Q

A 70-year-old man was admitted to the hospital 1 week ago with acute decompensated heart failure. Currently, he can walk about 200 yards before stopping because of fatigue and shortness of breath. Medical history is significant for nonischemic cardiomyopathy, with an implantable cardioverter-defibrillator placed 1 year ago. Medications are lisinopril, bumetanide, and spironolactone.

On physical examination, blood pressure is 90/70 mm Hg and pulse rate is 80/min. A grade 2/6 holosystolic murmur is heard at the left sternal border. No jugular venous distention is present, lungs are clear to auscultation, no S3 is heard, and there is no peripheral edema.

Electrocardiogram shows normal sinus rhythm with a QRS complex duration of 110 msec. Echocardiogram shows left ventricular enlargement, ejection fraction of 20%, and moderate mitral regurgitation but a structurally normal valve.

Which of the following is the most appropriate treatment?

A

Treatment should be started with a β-blocker, such as metoprolol succinate (long-acting form of metoprolol) or carvedilol. Patients with systolic heart failure should be treated with a β-blocker, regardless of symptom status, including heart failure that is asymptomatic or mildly symptomatic. However, β-blocker therapy generally should not be initiated in patients with decompensation, such as volume overload or a low-output state. Even patients with severe heart failure benefit from and tolerate β-blocker therapy. Even patients with significant pulmonary disease tolerate β-blocker therapy well, in particular, the more β1-selective agents, such as metoprolol or bisoprolol. Initiation of β-blocker therapy is usually tolerated when patients are clinically stable, before hospital discharge.

This patient likely has functional mitral regurgitation secondary to tethering of the mitral valve leaflets as a result of ventricular dilation in the setting of cardiomyopathy. No definitive evidence suggests that surgically correcting functional mitral regurgitation in the setting of nonischemic cardiomyopathy improves survival rates.

Aldosterone antagonists block the deleterious effects of aldosterone on the heart and have been shown to improve survival, decrease symptoms, and improve functional level in patients with advanced heart failure (class III-IV symptoms and left ventricular ejection fraction of ≤35%). Spironolactone is usually the first-line therapy based on clinical experience and cost considerations, although eplerenone may be useful in those who have gynecomastia as a result of treatment with spironolactone. This patient has tolerated spironolactone without difficulty; therefore, no change in aldosterone antagonist therapy is indicated.

Biventricular pacing, together with an implantable cardioverter-defibrillator (cardiac resynchronization therapy), is indicated for patients with severe symptomatic heart failure (New York Heart Association [NYHA] class III-IV), ejection fraction of less than 35%, and ventricular dyssynchrony, as evidenced by a prolonged QRS complex on electrocardiogram (>120 msec) while taking optimal medical therapy. This patient does not have a prolonged QRS complex and therefore would not benefit from upgrading to a biventricular device.

38
Q

A 67-year-old woman is evaluated because of a 1-month history of shortness of breath and lower extremity edema. She has no chest pain, palpitations, or syncope. Medical history is notable for hypertension. Her only medication is hydrochlorothiazide.

On physical examination, blood pressure is 170/90 mm Hg and pulse rate is 65/min. Estimated central venous pressure is 12 cm H2O. Cardiac examination shows a regular rhythm with no murmurs. Crackles are heard bilaterally in the lung bases. Pitting edema to the midshin is present bilaterally.

The electrocardiogram shows left ventricular hypertrophy by voltage criteria but no evidence of previous myocardial dysfunction. Echocardiogram shows moderate concentric left ventricular hypertrophy and no significant valvular abnormalities. Left ventricular ejection fraction is 60%.

Hydrochlorothiazide is discontinued, and treatment with furosemide is started.

Which of the following is the most appropriate additional therapy for this patient?

A

The most appropriate treatment for the patient is an angiotensin receptor blocker, such as candesartan. She has heart failure with preserved ejection fraction (HFPEF, or diastolic heart failure), as evidenced by signs and symptoms of heart failure (dyspnea, edema, examination findings consistent with volume overload) in the setting of a normal ejection fraction, as measured by echocardiography.

There is not a large body of evidence to guide the treatment of HFPEF. Therefore, treatment is primarily focused on managing the manifestations of heart failure (volume overload) and targeting risk factors for left ventricular hypertrophy (primarily hypertension), which is strongly associated with HFPEF. This patient’s hypertension is not well controlled. In addition to reducing congestion by increasing the dose of diuretics, an antihypertensive agent should be added.

The angiotensin receptor blocker candesartan is an agent that has been studied in a large randomized controlled trial of HFPEF treatment and was associated with a reduction in hospitalizations. Other appropriate agents for use in HFPEF include angiotensin-converting enzyme inhibitors, nondihydropyridine calcium channel blockers (verapamil, diltiazem), and β-blockers.

Patients with HFPEF tend to have a small, stiff left ventricle that may be susceptible to excessive preload reduction that can lead to ventricular underfilling and decreased cardiac output. Therefore, medications that act primarily by decreasing preload are usually avoided, such as nitrates and dihydropyridine calcium channel blockers. Therefore, the preferred therapy would not include the calcium channel blocker amlodipine or the long-acting nitrate isosorbide mononitrate.

Although digoxin may be used in patients with severe systolic heart failure, there is no established role for digoxin in the treatment of HFPEF. This agent is generally avoided because of its positive inotropic effect.

39
Q

A 35-year-old woman is evaluated for follow-up of heart failure secondary to peripartum cardiomyopathy. Symptoms began in the third trimester, and therapy was initiated at that time. The patient gave birth to a healthy baby 3 weeks ago but remains symptomatic. She can walk approximately one block on level ground. Medical history is otherwise unremarkable. Medications are carvedilol, spironolactone, and hydrochlorothiazide.

On physical examination, the patient is afebrile, blood pressure is 100/70 mm Hg, and pulse rate is 50/min. No jugular venous distention is present. Lungs are clear to auscultation. Cardiac examination shows a regular rhythm and a normal S1 and S2, without an S3. No peripheral edema is present.

Electrocardiogram shows normal sinus rhythm with a QRS complex duration of 110 msec and nonspecific ST-segment changes. Echocardiogram shows left ventricular ejection fraction of 25%.

Which of the following is the most appropriate management?

A

The patient has systolic heart failure as a result of peripartum cardiomyopathy. Treatment with enalapril should be started. The patient appears euvolemic and reports symptoms of New York Heart Association (NYHA) functional class III heart failure. Medical therapy for heart failure during pregnancy should include standard therapy for heart failure with a β-blocker and diuretic. However, New York Heart Association inhibitors and angiotensin receptor blockers should be excluded until after delivery. Treatment with an ACE inhibitor, such as enalapril, should therefore be initiated. Because the patient has severe symptoms (NYHA class III), treatment with an aldosterone antagonist (spironolactone) is also indicated.

Cardiac resynchronization therapy is indicated in patients with heart failure (ejection fraction <35%) with persistent moderate to severe symptoms despite optimal medical therapy and a QRS interval of greater than 120 msec. However, the patient’s medical therapy is not optimal because she is not taking an ACE inhibitor and her QRS interval is not prolonged.

Endomyocardial biopsy is generally indicated to assist in the diagnosis of suspected infiltrative or inflammatory cardiomyopathy in which a definitive diagnosis would affect treatment or prognosis, such as with amyloidosis, hemochromatosis, or sarcoidosis. The patient does not have significant evidence to suggest inflammatory cardiomyopathy, such as increased ventricular wall thickness; infiltrative cardiomyopathy, such as low voltage on electrocardiogram; or systemic manifestations of illness, such as fever.

Although, in general, it is desirable to titrate the β-blocker to target doses, increasing the dose of carvedilol in this patient would likely be limited by her relatively slow heart rate and significant fatigue.

40
Q

A 55-year-old man is evaluated during a routine examination. He has a 2-year history of nonischemic cardiomyopathy, with echocardiogram showing left ventricular ejection fraction of 35%. He is feeling well and reports no shortness of breath; he walks 2 miles daily without symptoms. Medical history is remarkable for hypertension. Medications are lisinopril, carvedilol, and chlorthalidone.

On physical examination, blood pressure is 150/90 mm Hg and pulse rate is 50/min. No jugular venous distention is present. Lungs are clear to auscultation. Cardiac examination shows a regular rhythm with no murmurs or gallops. No edema is present.

Laboratory studies show serum creatinine level of 1.5 mg/dL (133.0 µmol/L), sodium level of 138 meq/L (138 mmol/L), and potassium level of 4.0 meq/L (4.0 mmol/L).

Electrocardiogram shows a normal sinus rhythm and left ventricular hypertrophy.

Which of the following calcium channel blockers should be added to this patient’s medical regimen?

A

A patient with resistant hypertension (blood pressure that is not at the target value with three-drug therapy with different classes of drugs, including a diuretic) and systolic heart failure should begin taking a newer-generation dihydropyridine calcium channel blocker, such as amlodipine, to improve control of blood pressure.

Although specific combinations of drugs have not been well studied in patients with resistant hypertension, many experts recommend adding a newer-generation dihydropine calcium channel blocker to an angiotensin-converting enzyme (ACE) inhibitor and a diuretic when blood pressure is not at the target value. Large-scale clinical trials have shown amlodipine and felodipine to have little or no negative inotropic effect and a neutral effect on morbidity and mortality rates. However, because they do not decrease morbidity or mortality rates, calcium channel blockers are not first-line treatment for systolic heart failure. Use of calcium channel blockers in systolic heart failure is generally reserved for treatment of conditions such as hypertension or angina that are not optimally managed with maximal doses of evidence-based medications such as ACE inhibitors and β-blockers.

Many calcium channel blockers are relatively contraindicated in patients with systolic heart failure because of an associated increased risk of exacerbation of heart failure. Older-generation calcium channel blockers, such as diltiazem, nifedipine, and verapamil, may precipitate exacerbation of heart failure because of their negative inotropic effects. Although this patient’s symptoms are currently controlled (New York Heart Association class I), he would still be at risk for exacerbation of heart failure with one of these agents.

41
Q

A 63-year-old man is evaluated because of increased dyspnea and wheezing of 5 days’ duration. New lower extremity edema has also developed over the last 6 weeks.

On physical examination, the patient is afebrile, blood pressure is 162/92 mm Hg, pulse rate is 116/min, respiration rate is 24/min, and oxygen saturation is 90% on ambient air. He has jugular venous distention and an early 2/6 systolic murmur isolated to the lower left sternal border. The murmur increases in intensity with inspiration. The liver is enlarged to palpation. Pedal edema is present bilaterally.

Which of the following is the most likely diagnosis?

A

The most likely diagnosis in this patient is tricuspid regurgitation. Tricuspid regurgitation is characterized by a systolic murmur at the lower left sternal border that may increase in intensity with inspiration. The murmur does not radiate well, although it can sometimes be heard at the upper left sternal border. A finding characteristic of right-sided murmurs is augmentation in intensity with inspiration. Tricuspid regurgitation most often is caused by left-sided heart disease that causes pulmonary hypertension, which leads to right ventricular enlargement and annular dilation. Primary pulmonary hypertension and elevated pulmonary pressure as a result of chronic lung disease also cause tricuspid regurgitation, with the term cor pulmonale describing right-sided heart failure as a result of pulmonary hypertension in the absence of left-sided heart disease. Other causes of tricuspid regurgitation include endocarditis, injury after pacer lead placement, carcinoid disease, mediastinal irradiation, and trauma. Carcinoid disease causes direct toxicity to the tricuspid valve, seen as leaflet thickening and retraction. In mild or moderate tricuspid regurgitation, most patients are asymptomatic. Severe tricuspid regurgitation may be associated with signs of advanced right-sided heart failure, including jugular venous distention, ascites, hepatomegaly (sometimes pulsatile), and lower extremity edema. Electrocardiography may show enlargement of the right atrium or the right ventricle.

Auscultatory features of aortic stenosis include a mid- to late-peaking systolic murmur that radiates to the carotid arteries, an S4, a single S2 as a result of loss of the aortic closure component, and delayed timing and decreased amplitude in the carotid pulses (pulsus parvus et tardus). Physical examination findings of chronic mitral regurgitation in patients with hyperdynamic systolic function include a prominent apical impulse with normal pulse pressure. The murmur is usually holosystolic, is heard best at the apex, and radiates laterally or posteriorly. The murmur of pulmonic regurgitation is diastolic murmur.

42
Q

A 72-year-old man is evaluated because of progressive dyspnea that began 3 weeks ago. Six years ago, he underwent replacement of the aortic valve with a prosthetic value for treatment of calcific aortic stenosis. Until 3 weeks ago, he had been doing well. He has no other medical problems, and his only medication is warfarin.

On physical examination, he is afebrile, blood pressure is 134/68 mm Hg, pulse rate is 88/min, respiration rate is 20/min, and oxygen saturation is 96% on ambient air. Cardiac examination shows a laterally displaced apex beat, regular S1 and S2, a sharp aortic valve click, expiratory splitting of the S2, a 2/6 systolic murmur that is loudest at the right second intercostal space without radiation, and a 1/6 blowing diastolic murmur that is loudest at the third left intercostal space. The remainder of the physical examination is normal.

Chest radiography shows cardiomegaly without interstitial edema or pulmonary vascular congestion. Electrocardiogram shows left bundle branch block.

Which of the following physical examination findings most strongly suggests dysfunction of the prosthetic heart value?

A

Development of a new diastolic murmur of aortic regurgitation is most closely associated with prosthetic value dysfunction. The murmur of aortic regurgitation is a soft, blowing diastolic murmur that is often heard best at the third left or second right intercostal space. It does not radiate well and may be confined to a very limited area of the chest wall. The murmur can be heard best with the patient leaning forward in end-expiration. A sharp valve click is often heard on auscultation of normal prosthetic valves. Dampening of a previously sharp valve click can suggest a thrombus or vegetation on the valve. Most individuals with prosthetic aortic valves have a systolic heart murmur because of turbulence across the prosthesis. This murmur is physiologic and does not indicate valvular dysfunction. Expiratory splitting of S2 may be heard in patients with left bundle branch block or severe left ventricular hypertrophy as a result of delayed emptying of the left ventricle.

43
Q

A 38-year-old man is evaluated during a routine health examination. He exercises 2 or 3 days each week by jogging for 30 minutes without shortness of breath or chest discomfort. During stressful emotional situations, he occasionally feels “skipped heart beats” but has not had prolonged palpitations, presyncope, or syncope. He generally feels healthy. He has no history of medical problems and takes no medications. He has not had fever or chills.

Physical examination shows normal temperature, blood pressure of 124/68 mm Hg, pulse rate of 64/min and regular, and respiration rate of 14/min. Cardiac examination shows a grade 2/6 early systolic crescendo-decrescendo murmur heard at the lower left sternal border without radiation. The lungs are clear, and the peripheral pulses are normal.

Electrocardiogram shows normal results.

Which of the following is the most appropriate next test?

A

No additional testing is needed for this patient. He has an asymptomatic benign systolic ejection murmur. The benign characteristics of the murmur include its intensity or grade (<3/6), timing (early and brief systolic), lack of radiation, and absence of additional abnormal heart sounds. The remainder of the findings on physical examination and electrocardiogram are normal, without evidence of cardiac enlargement or dysfunction. In this common situation, the patient should be reassured. No additional diagnostic testing is indicated.

Ambulatory electrocardiography, obtained either continuously for 24 to 48 hours or as event-activated recordings, is not indicated. The patient’s brief episodes of palpitations are sporadic and are not associated with hemodynamic abnormalities. In patients with repetitive, frequent palpitations, ambulatory electrocardiography may be diagnostically useful.

Transesophageal echocardiography may be useful in patients in whom a transthoracic study does not provide adequate diagnostic information or to evaluate the feasibility of surgical repair when surgery is planned. However, this testing is not indicated in this patient.

Transthoracic echocardiography is recommended for diagnosis of systolic murmurs that are grade 3/6 or greater in intensity, diastolic murmurs, continuous murmurs, holosystolic murmurs, late systolic murmurs, murmurs associated with ejection clicks, or murmurs that radiate to the neck or back. This patient’s murmur does not have any of these characteristics.

44
Q

A 72-year-old man is evaluated in the emergency department because of worsening shortness of breath, orthopnea, and lower extremity edema. He has chest heaviness with exertion, but no presyncope or syncope.

On physical examination, blood pressure is 118/74 mm Hg, pulse rate is 96/min, and respiration rate is 20/min. Estimated central venous pressure is 10 cm H2O. Cardiac examination shows a regular heart rate and S2 that is diminished in intensity. A grade 3/6 systolic murmur at the left lower sternal border and an S3are present. Lung examination shows bibasilar crackles, and bilateral lower extremity edema of the knees is present.

Chest radiograph shows cardiomegaly and increased bilateral interstitial markings. Electrocardiogram shows sinus rhythm and left ventricular hypertrophy. Transthoracic echocardiogram shows concentric left ventricular dilation, with an ejection fraction of 30%. The aortic valve leaflets are calcified, with reduced mobility. The calculated valve area is 0.8 cm2, and the estimated transvalvular pressure gradient is elevated.

A

Aortic valve replacement surgery is the appropriate treatment for the patient. He has decompensated heart failure caused by severe aortic stenosis. The aortic valve is calcified with a significantly decreased valve area. Despite severe left ventricular systolic dysfunction (as seen by the ejection fraction of 30%), the increased transvalvular pressure gradient suggests that his cardiac output is preserved and that the stenotic valve is the cause of the patient’s heart failure. In patients with severe valve dysfunction with symptoms or abnormal ventricular function, surgical valve repair or replacement is the only definitive intervention.

Balloon aortic valvuloplasty is indicated only for patients with severe aortic stenosis with hemodynamic compromise or deterioration as a bridge to eventual aortic valve replacement. Although this patient has symptoms of heart failure, he is hemodynamically stable and has improved quickly with diuretic therapy. Balloon aortic valvuloplasty for calcific aortic stenosis results in only a small improvement in aortic valve area. It is associated with a high risk of stroke and has a high rate of restenosis 6 months after the procedure.

Patients with severe aortic stenosis complicated by decompensated heart failure and low cardiac output may benefit from intravenous nitroprusside. However, this patient does not have hemodynamic instability, and his response to intravenous diuretic therapy has not been assessed. Therefore, the use of nitroprusside is not indicated.

Transcatheter aortic valve implantation (TAVI) is a novel therapy for aortic valve replacement in patients with a very high predicted risk of operative mortality. Early studies of TAVI in patients with very high operative risk (approaching 50%) have shown improved survival compared with medical therapy, including balloon aortic valvuloplasty. However, this patient has an acceptable operative risk for aortic valve replacement and does not require TAVI.

45
Q

A 43-year-old woman is evaluated because of a 3-month history of substernal exertional chest pain and worsening dyspnea on exertion. Medical history is significant for Hodgkin lymphoma diagnosed 15 years ago that was treated with radiation therapy that involved the thorax.

On physical examination, temperature is normal, blood pressure is 148/41 mm Hg, pulse rate is 80/min, and respiration rate is 14/min. Carotid upstrokes are rapid and accentuated, with a rapid decline. She has no jugular venous distention. The S2 is diminished in intensity, and there is no S3 gallop present. A grade 2/6 high-pitched blowing diastolic decrescendo murmur is heard to the left of the sternum at the third intercostal space. The apical point of maximal impulse is displaced inferiorly and laterally. There is no hepatojugular reflux.

Which of the following is the most likely cause of the patient’s symptoms?

A

The most likely cause of this patient’s chest pain and worsening dyspnea is aortic valve regurgitation. On physical examination, the patient has a diastolic murmur, which is a key to diagnosis. Additionally, the carotid arteries have a rapid, accentuated upstroke, with a rapid decline (frequently referred to as a Corrigan pulse); the point of maximal impulse is displaced (suggesting left ventricular volume overload); and the pulse pressure is widened (systolic pressure minus diastolic pressure; normal is ≤40 mm Hg). These findings highly support a diagnosis of aortic regurgitation.

Aortic valve disease as a result of previous radiation therapy is the most likely cause of aortic regurgitation in this patient. The risk of cardiotoxicity, including valvular fibrosis and regurgitation, increases with higher total radiation dose. However, there is no single dose of radiation below which cardiotoxicity will not occur. The clinical onset of radiation-induced valvular regurgitation is variable and may occur 10 to 25 years or more after initial radiation therapy to the thorax. Clinically significant aortic valve regurgitation may occur in 25% or more of patients with previous radiation to the thorax. In this patient, dyspnea can be explained on the basis of elevation in left ventricular diastolic pressure as a result of hemodynamically significant aortic regurgitation. Chest pain occurs as a result of low coronary filling pressures and subsequent myocardial ischemia caused by low diastolic aortic pressure induced by aortic regurgitation.

Constrictive pericarditis should be considered in a patient with previous radiation of the thorax who presents with dyspnea on exertion. The right ventricle is more extensively involved, typically leading to right ventricular failure (jugular venous distention, hepatojugular reflux, peripheral edema). Physical examination in this patient showed no right ventricular involvement, making constrictive pericarditis unlikely.

Restrictive cardiomyopathy may occur as a result of previous radiation therapy but is unlikely in this patient. Findings on physical examination do not indicate right-sided pressure overload (jugular venous distention, hepatojugular reflux), which would be expected in a patient with symptomatic restrictive cardiomyopathy.

Tricuspid regurgitation also may occur as a result of previous radiation therapy to the thorax. However, findings on physical examination do not show evidence of hemodynamically significant tricuspid regurgitation, such as jugular venous distention, large retrograde v waves, or hepatojugular reflux. In severe tricuspid regurgitation, a systolic murmur is usually present but may be absent. When absent, however, a large jugular vein v wave would be expected.

46
Q

A 43-year-old man is evaluated during a routine examination. He is in good health and exercises regularly without difficulty. He has no history of presyncope, syncope, or palpitations. He is a nonsmoker and takes no medications.

On physical examination, he is afebrile, blood pressure is 120/64 mm Hg, pulse rate is 80/min and regular, and respiration rate is 16/min. Cardiac examination shows a normal S1 and a physiologically split S2. There is a grade 2/6 decrescendo diastolic murmur at the left sternal border. Distal pulses are normal, and there is no pedal edema.

Transthoracic echocardiogram shows normal ventricular size and function, with an ejection fraction of 65%. There is a bicuspid aortic valve with moderate regurgitation. The diameter of the proximal ascending aorta is normal, as are the estimated pulmonary arterial pressures.

Which of the following is the most appropriate next step in the management of this patient?

A

Clinical follow-up in 1 year is appropriate management for this patient. A bicuspid aortic valve is the most common congenital heart valve abnormality. It affects 1% of the population, with a male predominance. Because a bicuspid valve is subject to higher mechanical and shear stress than other valves, the disease process of progressive calcification is accelerated and clinical presentation tends to be earlier, in the fourth or fifth decade of life. A bicuspid aortic valve may have significant aortic regurgitation or stenosis and is at risk for infective endocarditis. A bicuspid aortic valve may also be associated with an underlying disorder of vascular connective tissue, with a loss of elastic tissue that leads to progressive dilation of the ascending aorta. Approximately 50% of patients have aortic root or ascending aortic dilation. This patient has a bicuspid aortic valve with moderate aortic regurgitation but normal left ventricular size and systolic function. Pulmonary pressures are in the normal range, and there is no evidence of adverse hemodynamic effects of valve regurgitation on the ventricle. Therefore, the patient needs no additional diagnostic studies or specific treatment at this time. However, because worsening of aortic regurgitation can be insidious, routine clinical follow-up is indicated at least yearly, typically with repeat transthoracic echocardiography to monitor for disease progression.

Aortic valve replacement surgery is recommended in patients with severe aortic regurgitation and cardiopulmonary symptoms. In asymptomatic patients with severe regurgitation, surgery is recommended when there are signs of adverse hemodynamic effects on the left ventricle, when there is left ventricular enlargement, or when the ejection fraction is less than 55%. This patient has moderate aortic regurgitation. He is asymptomatic and has normal left ventricular size and function. Valve replacement surgery is not indicated at this time. Patients with aortic root dilation should also be evaluated for surgery. If moderate dilation (diameter 4.0-4.9 cm) is noted, serial imaging is recommended to monitor for progressive dilation and dysfunction. Surgery is recommended when the aortic root or proximal ascending aorta exceeds 5 cm in diameter.

Transthoracic echocardiography is generally adequate for visualization of the proximal aortic root and routine evaluation and follow-up of a bicuspid aortic valve. However, transesophageal echocardiography or cardiac magnetic resonance imaging or computed tomography may be useful if inadequate visualization with transthoracic imaging is not possible.

47
Q

A 60-year-old woman is evaluated because of progressive exertional dyspnea and shortness of breath over the last 6 months. She has been healthy and has had no chest pain or other cardiovascular symptoms. Medical history is unremarkable. She is a nonsmoker and takes no medications.

On physical examination, the patient is afebrile, blood pressure is 122/85 mm Hg, and pulse rate is regular at 90/min. Estimated central venous pressure is 10 cm H2O. Lungs are clear to auscultation. Cardiac examination shows a parasternal and prominent apical impulse, a loud S1, a normal S2, and an opening snap. There is a grade 2/6 holosystolic murmur at the cardiac apex radiating to the axilla and a low-pitched 3/6 middiastolic murmur after the opening snap that accentuates presystole. Trace bipedal edema is present.

Electrocardiogram shows sinus rhythm, left atrial enlargement, and right axis deviation.

Which of the following is the most likely cause of this patient’s symptoms?

A

This patient most likely has mitral stenosis as the cause of her progressive shortness of breath. Mitral stenosis should be suspected because of the opening snap followed by a diastolic murmur that is accentuated with atrial contraction. S1 is usually loud; S2 may be variable in intensity. The parasternal lift supports the presence of right ventricular hypertrophy. Electrocardiogram in patients with mitral stenosis typically shows features of left atrial enlargement and hypertrophy and right axis deviation.

Aortic regurgitation causes a diastolic murmur that is best heard at the second right and third left interspaces; there is no opening snap or associated left atrial enlargement or right ventricular hypertrophy. Aortic regurgitation is also associated with a widened pulse pressure (systolic pressure minus diastolic pressure; normal is ≤40 mm Hg). These findings are not present in this patient.

A significant atrial septal defect produces a systolic murmur without an opening snap. Because of left-to-right blood flow, fixed splitting of the S2 and right-sided volume overload may result in right-sided chamber enlargement with a parasternal impulse. This patient’s diastolic murmur and other findings are not consistent with this diagnosis.

Patent ductus arteriosus causes a continuous murmur in the left parasternal location. In moderate to severe cases, blood flow from the aorta to the pulmonary artery may lead to increased pulmonary blood flow and left ventricular volume overload, with associated findings. Similar to aortic regurgitation, the carotid pulses are brisk as a result of increased stroke volume. Pulse pressure may be increased as a result of diastolic runoff into the pulmonary artery.

48
Q

A 54-year-old woman is evaluated because of progressive shortness of breath over the last 6 months. She has no chest pain. She has no history of cardiovascular disease or other medical problems. She is a nonsmoker and takes no medications. On physical examination, the patient is afebrile, blood pressure is 138/78 mm Hg, pulse rate is 95/min, and respiration rate is 14/min. Estimated central venous pressure is 8 cm H2O. Lung examination shows bibasilar crackles. Cardiac examination shows a grade 2/6 holosystolic murmur at the apex that radiates to the axilla and does not vary with respiration, a loud pulmonic component of S2, and an enlarged point of maximal impulse. Moderate bipedal edema is present.

Electrocardiogram shows normal sinus rhythm and left atrial and ventricular enlargement. Chest radiograph shows mild cardiomegaly and pulmonary congestion.

Which of the following is the most likely diagnosis?

A

This patient has findings consistent with chronic mitral regurgitation with associated hemodynamic and structural effects, ultimately leading to symptomatic heart failure. The location of the murmur is characteristic of mitral valve regurgitation (holosystolic, loudest at the apex, and radiating to the axilla). The prominent pulmonic component of the S2suggests pulmonary hypertension secondary to chronic mitral valve regurgitation. The patient has evidence of left-sided heart failure, including left atrial and ventricular enlargement on electrocardiogram, crackles on lung examination, and pulmonary congestion on chest radiograph. The signs of right-sided heart failure (peripheral edema, jugular venous distention) result from elevation of the left-sided filling pressures.

Chronic aortic valve stenosis can also cause left-sided heart failure, but the murmur of aortic stenosis is diamond-shaped, is loudest at the right sternal border, and radiates to the carotid arteries.

Mitral valve stenosis is characterized by an opening snap after the S2, followed by a low-frequency decrescendo murmur (diastolic “rumble”). Significant mitral valve stenosis causes elevated left atrial pressure, secondary pulmonary hypertension, and ultimately, right-sided heart failure. Because the left ventricle is protected from pressure or volume overload, mitral stenosis does not lead to left ventricular hypertrophy.

The murmur of aortic valve regurgitation is an early blowing diastolic murmur that is loudest at the left sternal border. Chronic aortic valve regurgitation may lead to left ventricular enlargement and left-sided heart failure.

The murmur of tricuspid valve regurgitation is systolic, is loudest at the lower left sternal border, and becomes louder with inspiration. Significant tricuspid valve regurgitation leads to right-sided heart failure but does not cause signs and symptoms of left-sided heart failure.

49
Q

A 19-year-old woman is evaluated because of palpitations. She describes the sensation of isolated “extra beats” that do not occur with any regularity. She has no personal or family history of cardiovascular disease, and she has not had either presyncope or syncope. She is a nonsmoker and takes no medications.

On physical examination, vital signs are normal. The lungs are clear. There are no extra heart sounds. There is a grade 2/6 late systolic murmur that is heard best at the cardiac apex and radiates toward the left axilla. A midsystolic click is heard. After a Valsalva maneuver and a squat-to-stand maneuver, the midsystolic click moves closer to the S1, but the intensity of the murmur does not change. The remainder of the examination is unremarkable.

Which of the following is the most likely cause of the heart murmur?

A

The patient’s heart murmur is most likely caused by mitral valve prolapse. The auscultatory features of mitral valve prolapse include a “click-murmur” complex. This complex includes a midsystolic click, believed to be caused by sudden tensing of the mitral subvalvular apparatus as the leaflets prolapse into the left atrium, followed by a late systolic murmur. Performing the Valsalva maneuver and standing from a squatting position decrease end-diastolic volume and move the click-murmur complex closer to the S1.

Mitral valve prolapse occurs in approximately 2% of the general population and is the most common cause of mitral regurgitation. In the absence of significant mitral regurgitation, primary mitral valve prolapse is usually asymptomatic but can present with palpitations or atypical chest discomfort. Palpitations are common and are usually associated with benign premature atrial or ventricular contractions. Sustained arrhythmias are exceedingly rare.

Hypertrophic cardiomyopathy is associated with a harsh crescendo-decrescendo systolic murmur that begins slightly after S1 and is heard best at the apex and lower left sternal border. Performing the Valsalva maneuver and standing from a squatting position increase the intensity of the murmur. The murmur of hypertrophic cardiomyopathy is the only murmur that increases in intensity with the Valsalva maneuver.

Benign (innocent) flow murmurs are typically midsystolic grade 1 to 2/6 murmurs associated with normal heart sounds and no other findings. The presence of a click, an S4, abnormal splitting of S2, or increased intensity or duration of the murmur on performing the Valsalva maneuver or standing from a squatting position is not compatible with a benign (innocent) flow murmur.

The murmur of mitral valve regurgitation begins shortly after S1 and ends just before S2 (holosystolic murmur). It is not associated with clicks, and the intensity is not increased with standing from a squatting position or performing the Valsalva maneuver.

50
Q

A 21-year-old man is evaluated during a routine medical examination. He feels well and is physically active. He plays sports and runs several times each week. He has no medical problems, does not smoke, and takes no medications or drugs.

On physical examination, blood pressure is 128/73 mm Hg, pulse rate is 56/min, and respiration rate is 16/min. There is no jugular venous distention. Carotid upstrokes are normal. There is a grade 2/6 early systolic murmur along the left lower sternal border that is accentuated by a Valsalva maneuver and is decreased with a handgrip maneuver. An S4 is also noted.

Electrocardiogram shows sinus bradycardia and left ventricular hypertrophy by voltage. Echocardiogram shows left ventricular hypertrophy with septal hypertrophy, a small left ventricular cavity, normal systolic function with an ejection fraction of 65%, and left atrial enlargement.

Which of the following is the most likely diagnosis?

A

The most likely diagnosis is hypertrophic cardiomyopathy. The findings on cardiac examination are consistent with dynamic left ventricular outflow tract obstruction, with a systolic murmur that is accentuated during maneuvers that decrease preload (Valsalva maneuver) but is attenuated by increasing afterload (handgrip maneuver). Echocardiographic findings confirm asymmetric septal hypertrophy that is consistent with hypertrophic cardiomyopathy.

The echocardiographic features in hypertrophic cardiomyopathy are diverse but include left ventricular hypertrophy. The hypertrophy may be concentric (particularly if marked), but may also disproportionately involve the septal, anterior, lateral, or apical walls. Dynamic left ventricular outflow tract or midcavity obstruction is a feature of hypertrophic cardiomyopathy. However, it is not always seen nor is it a necessary finding to confirm the diagnosis. Additional echocardiographic features include a small left ventricular cavity and significant left atrial enlargement. Although patients with hypertrophic cardiomyopathy may present with symptoms such as dyspnea, chest pain, or dizziness, many are asymptomatic. However, because of the increased risk of associated mortality and the genetic basis of the disease, diagnosis is critical, even in asymptomatic patients.

Athlete’s heart is concentric left ventricular hypertrophy that typically occurs in endurance athletes and sometimes in weightlifters. Concentric left ventricular hypertrophy may also be caused by long-standing hypertension, which is not present in this patient. Echocardiography is useful in differentiating hypertrophic cardiomyopathy from these other conditions. Compared with hypertrophic cardiomyopathy, athlete’s heart is more likely to show less marked hypertrophy, hypertrophy that is concentric without significant asymmetry, an enlarged left ventricular cavity, a lack of marked left atrial enlargement, and normal diastolic function. Cardiovascular magnetic resonance imaging is helpful for diagnosis if hypertrophic cardiomyopathy cannot be confirmed or differentiated from other causes.

Dilated cardiomyopathy is easily excluded on the basis of echocardiography, which does not show an enlarged left ventricle with systolic dysfunction (ejection fraction <40%), as would be expected with this diagnosis.

Restrictive cardiomyopathy could explain the finding of left ventricular hypertrophy. However, an accentuated rate of early diastolic filling (restrictive filling) is characteristic of this entity, and not impaired early filling, as is present in this patient. Lack of this pattern of filling virtually excludes restrictive cardiomyopathy.

51
Q

A 58-year-old man is evaluated for 3 to 4 months of progressive aching pain in the left buttock and hip. Pain occurs during walking and is relieved by rest. The patient has been experiencing erectile dysfunction over a similar period. He has a 30-pack-year history of smoking and quit 1 year ago. He has hypercholesterolemia and type 2 diabetes mellitus. Medications are aspirin, lisinopril, simvastatin, metformin, and metoprolol.

On physical examination, temperature is normal, blood pressure is 112/72 mm Hg, pulse rate is 68/min, and respiration rate is 16/min. Femoral, popliteal, and foot pulses are diminished. There is no distal ulceration or skin breakdown. Ankle-brachial index on the left side is 0.7.

Which of the following is the most likely site of this patient’s arterial disease?

A

This patient most likely has aortoiliac disease. Symptoms often give important clues as to the likely site of peripheral arterial disease. The patient has buttock and hip claudication, diminished femoral pulses, and erectile dysfunction, sometimes referred to as Leriche syndrome. This presentation most commonly represents atherosclerotic disease within the aortoiliac system.

Claudication as a result of aortoiliac disease often results in greater disability compared with more distal disease. Additionally, aortoiliac disease increases the risk of distal embolization. Accordingly, a more aggressive approach to aortoiliac disease is typically taken that may include either endovascular intervention or aortoiliac surgery.

Common femoral arterial occlusive disease may cause thigh pain with effort, but it would not result in erectile dysfunction. Because of the location of the common femoral artery with respect to the hip joint, surgical therapy or angioplasty, but not stenting, would be considered as part of therapy.

Occlusive disease within the popliteal artery would produce pain within the lower calf. Disease in this location should be managed primarily with an exercise program and medical therapy. Patients who do not benefit from such conservative management should be considered for femoral-popliteal bypass.

Occlusive disease within the superficial femoral artery usually produces effort-related discomfort in the upper calf. Angioplasty may be appropriate for patients who have symptoms related to the superficial femoral artery and have not benefited from medical therapy or who are extremely limited in activity due to ischemia despite medical therapy.

52
Q

A 46-year-old man is evaluated in the emergency department because of acute onset of severe chest and back pain 1 hour ago. The pain has been unremitting. Medical history is significant for a bicuspid aortic valve that was detected in adolescence because of a murmur and also for hypertension. His only medication is valsartan.

On physical examination, the patient is conscious and in pain. He is afebrile, blood pressure is 90/50 mm Hg, pulse rate is 118/min and regular, and respiration rate is 18/min.

Pulse rate and blood pressure are equal in both arms. Heart sounds are distant, regular, and rapid. A very faint early diastolic murmur is heard at the right upper sternal border. Estimated central venous pressure is 12 cm H2O.

Electrocardiogram indicates sinus tachycardia. A chest computed tomographic scan with intravenous contrast is shown :.

Which of the following is the most appropriate next step in management?

A

The computed tomographic scan with intravenous contrast shows : a dissection plane (arrows) extending from the proximal aorta through the arch (AA) and into the descending aorta (DAo). This is a Stanford type A dissection (involving the ascending aorta), which requires emergency surgical intervention.

The patient has a long-standing history of a murmur associated with a documented bicuspid aortic valve. The most frequent cardiovascular finding associated with a bicuspid valve is dilation of the proximal ascending aorta, which is related to abnormalities of the aortic media. These changes in the aortic media are independent of the degree of functional stenosis of the valve and increase the risk of aneurysm formation and aortic dissection.

Abrupt onset of severe chest and back pain is typical of an acute aortic syndrome. A diastolic murmur consistent with aortic valvular insufficiency increases the clinical suspicion for a proximal aortic dissection that has disrupted normal valve function. Although marked asymmetry in upper extremity pulses and pressures are classic findings in aortic dissection, in many cases, these features may not be present. Syncope occurs in approximately 10% of patients with an acute aortic dissection and is more commonly associated with proximal dissection. Syncope is associated with a higher coincidence of pericardial tamponade and worse in-hospital survival rates.

Further diagnostic imaging with coronary angiography, magnetic resonance imaging, or transesophageal echocardiography is not necessary and would only delay necessary surgical repair.

There is no accepted role for endovascular treatment of an acute Stanford type A aortic dissection.

53
Q

A 70-year-old man is evaluated in the emergency department for severe lower back pain that began suddenly 2 days ago and was associated with an episode of syncope. Since that time, he has had vague lower abdominal and back discomfort. He has had no change in bowel or urinary habits and no fever or chills. Medical history is significant for hypertension, hyperlipidemia, and a 40-pack-year smoking history. Medications are atorvastatin, aspirin, and lisinopril.

On physical examination, temperature is 37.7°C (99.8°F), blood pressure is 100/60 mm Hg, pulse rate is 98/min and regular, and respiration rate is 18/min. Results of cardiac and neurologic examinations are normal. Abdominal examination shows moderate tenderness to palpation in the infraumbilical and suprapubic regions, but without guarding or rebound tenderness. Findings on rectal examination are unremarkable, with guaiac-negative stool.

Laboratory results include hematocrit of 32% and leukocyte count of 12,000/µL (12.0 × 109/L). Results of liver chemistry studies and urinalysis are normal. Electrocardiogram shows normal sinus rhythm and evidence of left ventricular hypertrophy. Plain abdominal radiograph shows no free air or air-fluid levels.

Which of the following is the most likely diagnosis?

A

The patient’s clinical presentation of severe abdominal or back pain with syncope, followed by vague discomfort, is typical for a ruptured abdominal aortic aneurysm (AAA) that has been locally contained, preventing immediate death. The sentinel event of sudden, severe back pain associated with loss of consciousness marks the occurrence of rupture. Symptoms after that time are likely caused by either local irritation and inflammation related to the rupture and hemorrhage or expansion of the aneurysm against adjacent structures. Leukocytosis and anemia are common. CT scan should be performed for diagnosis, and the aneurysm should be repaired emergently.

This patient has several risk factors for AAA, including the major risk of cigarette smoking. The incidence of AAA is higher in men than in women and in whites versus blacks, and it increases with age. Hypertension and hyperlipidemia probably contribute to the risk of AAA development to a lesser degree.

Contained rupture of AAA, when misdiagnosed, is most often mistaken for renal colic, acute myocardial infarction, or diverticulitis. Renal colic may produce severe pain in the lower back, flank, or groin. Typically, the pain waxes and wanes. It is unlikely that renal colic would present with syncope, and the normal finding on urinalysis also makes this diagnosis unlikely.

Acute myocardial infarction can be associated with syncope, and electrocardiogram is not always diagnostic, particularly if there are findings such as left ventricular hypertrophy, which may obscure subtle abnormalities. However, the presence of abdominal and lower back pain rather than chest pain makes this diagnosis less likely.

Diverticulitis presents with crampy abdominal pain, most commonly in the left lower quadrant, often associated with a change in bowel habits. Leukocytosis may be present. Syncope associated with the onset of pain would be a very unusual presentation for this entity.

54
Q

A 76-year-old man is evaluated because of an episode of left-handed weakness involving all five digits that occurred yesterday and gradually subsided over 3 hours. He has had two similar episodes in the last 2 weeks. Medical history is unremarkable. His only medication is a daily low-dose aspirin.

On physical examination, blood pressure is 156/78 mm Hg and pulse rate is 76/min and regular. Cardiac examination shows a right carotid bruit. Other findings on physical examination, including a neurologic evaluation, are normal.

Electrocardiogram shows normal sinus rhythm with no evidence of ischemia. Carotid duplex ultrasound shows 80% to 99% stenosis of the right internal carotid artery, which is confirmed by computed tomographic angiography. Magnetic resonance imaging of the brain shows a 5-mm infarct in the right middle cerebral artery distribution.

Which of the following will have the greatest effect in reducing the risk of recurrent stroke in this patient?

A

This patient should be referred for immediate endarterectomy of the right internal carotid artery. He has had an acute ischemic stroke caused by symptomatic high-grade carotid stenosis. The risk of recurrent stroke is approximately 26% over the next 2 years. Carotid endarterectomy is highly effective in reducing the risk of recurrent stroke (number needed to treat = 17) in the immediate poststroke period. With symptomatic carotid stenosis, the risk of recurrent stroke is 1% per day for the first 2 weeks after a stroke or transient ischemic attack; therefore, the greatest benefit is gained when the procedure is performed early.

Carotid stenting would be inappropriate for this patient. Clinical trials have shown that for the primary outcomes of stroke, myocardial infarction, and death, stenting and endarterectomy are not significantly different. However, stenting poses a greater risk of perioperative stroke than endarterectomy. Therefore, it is not recommended as a primary intervention for symptomatic carotid artery stenosis.

Clopidogrel has a role in medical therapy for patients who are not able to take aspirin. However, antiplatelet agents generally provide only a marginal benefit compared with surgery in reducing the risk of stroke in the acute poststroke period in patients with symptomatic high-grade carotid stenosis.

Acute anticoagulation in the context of acute stroke has been associated with increased mortality rates and poorer neurologic outcomes. Similarly, long-term anticoagulation with warfarin or another oral anticoagulant is beneficial only in patients whose stroke is caused by atrial fibrillation or another source of cardiogenic embolism. Because this patient’s stroke likely resulted from his high-grade carotid stenosis and he has no other indication for anticoagulation, treatment with warfarin would not be appropriate.

55
Q

A 27-year-old woman is evaluated during a follow-up visit for high blood pressure that was diagnosed 4 months after she began taking an oral contraceptive pill. She has stopped using the oral contraceptive pill, but her blood pressure has remained high. She feels well and takes no medications. Medical history is otherwise unremarkable.

On physical examination, blood pressure is 166/108 mm Hg and the heart rate is 76/min. No orthostasis is present and the other vital signs are normal. There is a bruit noted in the right epigastric region. The remainder of the examination is unremarkable.

Kidney function is normal, and urinalysis is unremarkable.

A Doppler ultrasound study shows evidence of stenosis of the right renal artery. A kidney angiogram is shown :.

A

Percutaneous transluminal kidney angioplasty is indicated for this patient with renovascular hypertension secondary to fibromuscular dysplasia, a nonatherosclerotic, noninflammatory renovascular disease. Renovascular hypertension as a result of fibromuscular dysplasia is most commonly caused by medial fibroplasia of the renal artery. On angiogram, the characteristic finding of fibromuscular dysplasia is the “string of beads” appearance of the involved artery, which is apparent on this patient’s angiogram. Fibromuscular dysplasia is a disease of unknown cause and usually involves the renal and carotid arteries. Hypertension caused by fibromuscular dysplasia is more common in women and usually affects patients between 15 and 30 years of age. Catheter-based kidney angiography is the most accurate method to diagnose this condition; revascularization with kidney angioplasty may be performed at the same time as diagnostic angiography. The young age of many patients with fibromuscular dysplasia, such as this 27-year-old woman, reduces the risk of complications from this procedure.

The high likelihood of both technical success and meaningful blood pressure improvement as a result of kidney angioplasty makes drug therapy in this young patient unnecessary at this time.

Surgical revascularization is not the first-line treatment for this patient, given the higher risk of morbidity. Surgery should be reserved for patients who do not respond to kidney angioplasty or who have arterial anatomy that is too complex for kidney angioplasty.

56
Q

A 52-year-old man is evaluated in the emergency department for a 5-day history of pain and swelling in the right leg. He has no history of venous thromboembolism.

On physical examination, temperature is 36.5°C (97.7°F), blood pressure is 120/75 mm Hg, pulse rate is 85/min, and respiration rate is 22/min. The right lower extremity is swollen. Cardiopulmonary examination shows clear lungs and tachycardia.

A right popliteal vein deep venous thrombosis is confirmed by venous duplex compression ultrasonography. The patient is given low-molecular-weight heparin (LMWH).

Which of the following is the most appropriate management of this patient’s transition to warfarin therapy?

A

The most appropriate management of this patient’s transition from parenteral low-molecular-weight heparin (LMWH) to warfarin therapy requires at least 5 days of overlap with LMWH and warfarin therapy and an international normalized ratio of 2 or more for 24 hours. Randomized clinical trials show that 5 to 7 days of treatment with unfractionated heparin is as effective as 10 to 14 days of treatment when transitioning to warfarin therapy. Shorter durations of parenteral anticoagulation in the transition to vitamin K antagonists have not been tested and theoretically could confer a higher risk of recurrent thromboembolism. Warfarin acts as an anticoagulant by impairing hepatic synthesis of vitamin K-dependent coagulation factors rather than by directly inhibiting the function of already synthesized factors. Therefore, once an appropriate warfarin dose is initiated, the onset of therapeutic anticoagulation is dictated by the half-life of the coagulation factors. If a patient is receiving an adequate warfarin dose, it takes at least 5 days for vitamin K-dependent factor activity levels to decrease sufficiently for therapeutic anticoagulation (INR of 2-3) to occur. Consequently, parenteral anticoagulant therapy with LMWH should be continued along with warfarin for at least 5 days and until a therapeutic INR of 2 or more for 24 hours is achieved to avoid an increased risk of recurrent thromboembolism.

57
Q

A 75-year-old woman is evaluated in the hospital because of a 1-day history of swelling of the right leg. Three days ago, she underwent nephrectomy for renal cell carcinoma. Her only medication is unfractionated heparin, 5000 units subcutaneously twice daily.

On physical examination, blood pressure is 130/75 mm Hg, pulse rate is 85/min, and respiration rate is 20/min. Weight is 80 kg (176.3 lb). The right lower extremity is swollen, warm, and tender to palpation of the calf. The nephrectomy incision shows no erythema or bleeding. The remainder of the findings on examination are normal.

Laboratory studies:

Hematocrit

29%

Platelet count

275,000/µL (275 × 109/L)

Creatinine

2.2 mg/dL (194.5 µmol/L)

Estimated glomerular filtration rate

23 mL/min/1.73 m2

Venous duplex ultrasonography shows a right lower extremity femoral and popliteal vein deep venous thrombosis.

In addition to cessation of subcutaneous unfractionated heparin administration, which of the following is the most appropriate treatment?

A

Intravenous unfractionated heparin (UFH) adjusted to achieve a therapeutic activated partial thromboplastin time is the most appropriate treatment for this patient, who recently underwent a major surgical procedure and has chronic kidney disease. UFH is primarily cleared by the reticuloendothelial system rather than the kidneys. Therefore, it is preferable to the other choices for acute therapy for deep venous thrombosis (DVT). UFH also has a short half-life and is completely reversible with protamine.

Low-molecular-weight heparins are metabolized primarily by the kidneys, making dosing difficult in patients with a low glomerular filtration rate. The risk of underdosing may lead to inadequate therapy. Overdosing may lead to an increased risk of bleeding, and the ability to reverse anticoagulation with these agents is limited. Although anti-Xa levels may be helpful in monitoring the anticoagulant level, low-molecular-weight heparin is usually avoided in patients with advanced kidney disease.

Fondaparinux is cleared exclusively by the kidneys. Therefore, it is contraindicated in patients with poor renal function (estimated glomerular filtration rate <30 mL/min/1.73 m2). In addition, fondaparinux is not reversible with protamine. Consequently, potential bleeding is much more difficult to treat. Although the anticoagulant effects of fondaparinux can be treated with recombinant human factor VIIa, this factor concentrate has been associated with increased risk of thromboembolism, an important limitation in a patient with recent VTE.

Warfarin is metabolized by the liver and therefore does not accumulate in patients with worsening kidney function. However, its anticoagulant activity is delayed in onset by at least 5 to 7 days and is initially associated with transient hypercoagulability. Therefore, initial anticoagulation with warfarin in patients with acute thromboembolism is always done concomitantly with a parenteral agent, such as UFH.

58
Q

A 55-year-old man is evaluated in the emergency department because of a 3-day history of swelling, pain, and erythema of the right leg. He is otherwise active and healthy and reports no recent immobilization, surgery, or cancer.

On physical examination, temperature is 38.2°C (100.8°F). Other vital signs are normal. Examination of the left leg shows warmth and circumscribed erythema and tenderness limited to the posterior tibial area. The circumference of the left leg is 1 cm greater than that of the right leg when measured 10 cm below the tibial tuberosity. There is no pitting edema.

Which of the following is the most appropriate next step in diagnosis?

A

The most appropriate next diagnostic test is a D-dimer assay. Several imaging procedures can exclude deep venous thrombosis (DVT), but the diagnostic goal is to use the most efficient, least invasive, and least expensive method with the fewest side effects. A D-dimer assay is a simple, relatively noninvasive test that has been shown to have a high negative predictive value, especially if suspicion for DVT is low. A clinical prediction tool, the Wells score, has been established to help the clinician assess the likelihood of DVT. Studies have shown that when there is low clinical suspicion (as in this patient) and a negative finding on D-dimer assay, DVT can be reliably excluded without the need for more invasive or complex imaging.

According to the Wells criteria, the following clinical variables are assigned 1 point each: active cancer, paralysis or recent plaster cast, recent immobilization or major surgery, tenderness along the deep veins, swelling of the entire leg, a difference in calf circumference of more than 3 cm compared with the other leg, pitting edema, and collateral superficial veins. Clinical suspicion that an alternative diagnosis is likely is assigned -2 points. Based on this system, the pretest probability of DVT is considered high in patients with a score of 3 or higher, moderate in patients with a score of 1 to 2, and low in patients with a score of 0 or lower. This patient’s Wells score is -2; therefore, the likelihood of DVT is low. This patient’s low-grade fever, circumscribed area of warmth, and tenderness localized to the posterior calf could represent cellulitis, a reasonable alternative to the diagnosis of venous thrombosis.

Doppler ultrasonography is the most commonly used diagnostic study to evaluate for DVT and would be the test of choice for further evaluation, if indicated. Venography, the traditional gold standard for diagnosis of DVT, is rarely performed today because it is invasive, uncomfortable for the patient, expensive, and complex to perform. Computed tomography (or magnetic resonance imaging) scan of the leg has not been substantially validated as a reliable diagnostic test for DVT.

59
Q

75-year-old man is evaluated in the hospital because of community-acquired pneumonia. He is bedbound. He has heart failure and hypertension. Medications are lisinopril and carvedilol.

On physical examination, temperature is 38.6°C (101.4°F), blood pressure is 110/65 mm Hg, pulse rate is 90/min, and respiration rate is 24/min. The patient has right lower lobe bronchial breathing and egophony.

Leukocyte count is 17,000/µL (17 × 109/L).

Chest radiograph shows right lower lobe consolidation.

Which of the following is the most appropriate venous thromboembolism prophylaxis in this patient?

A

The most appropriate treatment is low-dose subcutaneous unfractionated heparin. The patient is immobilized and has at least two major risk factors for venous thromboembolism (VTE). These are age older than 60 years and acute infectious illness. Therefore, he should receive pharmacologic VTE prophylaxis with low-dose unfractionated heparin for at least the duration of hospitalization.

Treatment with aspirin, 325 mg/d, is not appropriate prophylaxis in this medically ill patient who is at high risk for VTE. Aspirin, which is effective for prevention of stroke and myocardial infarction, has not been shown to be effective as prophylaxis for VTE. Appropriate options for this patient include low-dose subcutaneous unfractionated heparin, low-molecular-weight heparin, and fondaparinux.

Use of pneumatic compression devices would not be the optimal approach to VTE prophylaxis in this patient. The use of intermittent pneumatic compression devices is an effective form of VTE prophylaxis. However, data supporting their efficacy are almost exclusively limited to surgical patients, and these devices have not been shown to significantly reduce the incidence of pulmonary embolism. Furthermore, there are serious deficiencies with adherence to mechanical prophylaxis in routine care settings. Therefore, in patients without contraindications to its use, pharmacologic treatment is appropriate for VTE prophylaxis. Nonpneumatic graded external compression stockings have not been shown to be effective. They also increase the risk of skin complications and therefore should not be used for VTE prophylaxis.

Warfarin, 1 mg/d, is not appropriate VTE prophylaxis for this medical inpatient. Although an initial study of low-dose warfarin showed efficacy in preventing DVT in patients with central venous catheters, subsequent studies have not replicated these promising results. Low-intensity, fixed-dose warfarin has never been shown to be useful in VTE prophylaxis. In patients with total hip arthroplasty and knee arthroplasty, adjusted-dose warfarin (international normalized ratio 2-3) has been shown to be efficacious in preventing VTE. However, no studies of this regimen have been conducted in medical inpatients.